Уравнения на комбинаторику с решением

Содержание
  1. Комбинаторика — правила, формулы и примеры с решением
  2. Всё о комбинаторике
  3. Комбинаторные задачи с решением
  4. Пример №1
  5. Пример №2
  6. Пример №3
  7. Пример №4
  8. Пример №5
  9. Пример №6
  10. Пример №7
  11. Пример №8
  12. Пример №9
  13. Пример №10
  14. Пример №11
  15. Пример №12
  16. Пример №13
  17. Пример №14
  18. Пример №15
  19. Пример №16
  20. Правила суммы и произведения
  21. Пример №17
  22. Пример №18
  23. Пример №19
  24. Пример №20
  25. Пример №21
  26. Пример №22
  27. Пример №23
  28. Размещения и перестановки
  29. Пример №24
  30. Пример №25
  31. Пример №26
  32. Пример №27
  33. Пример №28
  34. Пример №29
  35. Пример №30
  36. Пример №31
  37. Комбинации и бином ньютона
  38. Пример №32
  39. Пример №33
  40. Пример №34
  41. Пример №35
  42. Пример №36
  43. Пример №37
  44. Пример №38
  45. Пример №39
  46. Элементы комбинаторики
  47. Арифметика случайных событий
  48. Пример №40
  49. Теорема сложения вероятностей несовместных событий
  50. Зависимые и независимые события. Условная и безусловная вероятности
  51. Пример №41
  52. Теорема умножения вероятностей
  53. Что такое комбинаторика
  54. Понятие множества
  55. Равенство множеств
  56. Подмножество
  57. Операции над множествами
  58. Комбинаторика и Бином Ньютона
  59. Схема решения комбинаторных задач
  60. Понятие соединения
  61. Правило суммы
  62. Правило произведения
  63. Упорядоченные множества
  64. Размещения
  65. Пример №42
  66. Пример №43
  67. Пример №44
  68. Пример №45
  69. Перестановки
  70. Пример №46
  71. Пример №47
  72. Пример №48
  73. Сочетания без повторений
  74. Вычисление числа сочетаний без повторений с помощью треугольника Паскаля
  75. Пример №49
  76. Пример №50
  77. Бином Ньютона
  78. Объяснение и обоснование Бинома Ньютона
  79. Свойства биномиальных коэффициентов
  80. Пример №51
  81. Пример №52
  82. Зачем нужна комбинаторика
  83. Правило суммы
  84. Пример №53
  85. Правило произведения
  86. Пример №54
  87. Пример №55
  88. Пример №56
  89. Пример №57
  90. Пример №58
  91. Пример №59
  92. Пример №60
  93. Конспект урока на тему «Решение комбинаторных уравнений» (10 класс)
  94. Математика — онлайн помощь
  95. ПРИМЕРЫ РЕШЕНИЯ ЗАДАЧ
  96. Примеры и задачи для самостоятельного решения
  97. 💥 Видео

Видео:Решите уравнение ➜ ДВИ до ЕГЭСкачать

Решите уравнение ➜ ДВИ до ЕГЭ

Комбинаторика — правила, формулы и примеры с решением

Комбинаторика — это раздел математики, в котором изучаются способы выбора и размещения элементов некоторого конечного множества на основании определенных условий. Выбранные (или выбранные и размещенные) группы элементов называются соединениями. Если все элементы полученного множества разные, получаем соединения без повторений, а если элементы повторяются — соединения с повторениями.

Содержание:

В комбинаторике перестановка — это упорядоченный набор без повторений чисел.

Перестановкой из n элементов называется любое упорядоченное множество из n данных элементов.

Иными словами, это такое множество, для которого указано, какой элемент находится на первом месте, какой — на втором, . какой — на n-м.

Формула числа перестановок Уравнения на комбинаторику с решением

Уравнения на комбинаторику с решением

Количество различных шестизначных чисел, которые можно составить из цифр 1, 2, 3, 4, 5, 6, не повторяя эти цифры в одном числе, равноУравнения на комбинаторику с решением

Размещением из n элементов по k называется любое упорядоченное множество из k элементов, состоящее из элементов данного n-элементного множества.

Формулы для нахождения количества соединений с повторениями обязательны только для классов физико-математического профиля.

Формула числа размещений Уравнения на комбинаторику с решением

Уравнения на комбинаторику с решением

Количество различных трехзначных чисел, которые можно составить из цифр 1, 2, 3, 4, 5, 6, если цифры не могут повторяться, равно

Уравнения на комбинаторику с решением

Сочетанием без повторений из n элементов по k называется любое k-элементное подмножество данного n-элементного множества.

Формула числа сочетаний Уравнения на комбинаторику с решением

Уравнения на комбинаторику с решением(по определению считают, чтоУравнения на комбинаторику с решением

Из 25 учащихся одного класса можно выделить пятерых для дежурства по школе Уравнения на комбинаторику с решениемспособами, то есть Уравнения на комбинаторику с решениемспособами.

Некоторые свойства числа сочетаний без повторений

Уравнения на комбинаторику с решением(в частности, Уравнения на комбинаторику с решением)

Уравнения на комбинаторику с решением

Схема поиска плана решения простейших комбинаторных задач:

Если элемент А можно выбрать т способами, а элемент В — n способами (при этом выбор элемента А исключает одновременный выбор элемента В), то А или В можно выбрать m + n способами.

Если элемент А можно выбрать m способами, а после этого элемент В — n способами, то А и В можно выбрать Уравнения на комбинаторику с решениемспособами.

Уравнения на комбинаторику с решением

Объяснение и обоснование:

Понятие соединения. Правило суммы и произведения:

При решении многих практических задач приходится выбирать из определенной совокупности объектов элементы, имеющие те или иные свойства, размещать их в определенном порядке и т. д. Поскольку в этих задачах речь идет о тех или иных комбинациях объектов, то такие задачи называют комбинаторными. Раздел математики, в котором рассматриваются методы решения комбинаторных задач, называется комбинаторикой. В комбинаторике рассматривается выбор и размещение элементов некоторого конечного множества на основании определенных условий.

Выбранные (или выбранные и размещенные) группы элементов называют соединениями. Если все элементы полученного множества разные, получаем размещения без повторений, а если элементы могут повторяться — размещения с повторениями. В этом параграфе мы рассмотрим соединения без повторений.

Решение многих комбинаторных задач базируется на двух основных правилах — правиле суммы и правиле произведения.

Правило суммы. Если на тарелке лежат 5 груш и 4 яблока, то выбрать один фрукт (грушу или яблоко) можно 9 способами (5 + 4 = 9). В общем виде справедливо такое утверждение:

  • если элемент А можно выбрать m способами, а элемент В — n способами (при этом выбор элемента А исключает одновременный выбор элемента В), то А или В можно выбрать m + n способами.

Уточним содержание этого правила, используя понятие множеств и операций над ними.

Пусть множество А состоит из m элементов, а множество В -из n элементов. Если множества А и В не пересекаются (то есть Уравнения на комбинаторику с решением), то множество А Уравнения на комбинаторику с решениемВ состоит изУравнения на комбинаторику с решениемэлементов.

Правило произведения. Если в киоске продают ручки 5 видов и тетради 4 видов, то выбрать набор из ручки и тетради (то есть пару — ручка и тетрадь) можно 5æ4 = 20 способами (поскольку с каждой из 5 ручек можно взять любую из 4 тетрадей). В общем виде имеет место такое утверждение:

  • если элемент А можно выбрать m способами, а после этого элемент В — n способами, то А и В можно выбрать Уравнения на комбинаторику с решениемспособами.

Это утверждение означает, что если для каждого из m элементов А можно взять в пару любой из n элементов В, то количество пар равно произведению Уравнения на комбинаторику с решением.

В терминах множеств полученный результат можно сформулировать следующим образом. Если множество А состоит из т элементов, а множество В — из n элементов, то множество всех упорядоченных пар* (а; b), где первый элемент принадлежит множеству А (а ∈ А), а второй  множеству В (b ∈ В), состоит из Уравнения на комбинаторику с решениемэлементов.

Повторяя приведенные рассуждения несколько раз (или, более строго, используя метод математической индукции), получаем, что правила суммы и произведения можно применять при выборе произвольного конечного количества элементов.

Упорядоченные множества:

При решении комбинаторных задач приходится рассматривать не только множества, в которых элементы можно записывать в любом порядке, но и так называемые упорядоченные множества. Для упорядоченных множеств существенным является порядок следования их элементов, то есть то, какой элемент записан на первом месте, какой на втором и т. д. В частности, если одни и те же элементы записать в разном порядке, то мы получим различные упорядоченные множества. Чтобы различить записи упорядоченного и неупорядоченного множеств, элементы упорядоченного множества часто записывают в круглых скобках, например (1; 2; 3) ≠ (1; 3; 2).

Рассматривая упорядоченные множества, следует учитывать, что одно и то же множество можно упорядочить по-разному. Например, множество из трех чисел можно упорядочить по возрастанию: (–5; 1; 3), по убыванию: (3; 1; –5), по возрастанию абсолютной величины числа: (1; 3; –5) и т. д.

* Множество всех упорядоченных пар (а; b), где первый элемент принадлежит множеству А (а ∈ А), а второй — множеству В (b ∈ В), называют декартовым произведением множеств А и В и обозначают А × В. Отметим, что декартово произведение В × А также состоит из m*n элементов.

Заметим следующее: для того чтобы задать конечное упорядоченное множество из n элементов, достаточно указать, какой элемент находится на первом месте, какой на втором, . какой на n-м.

Размещения:

Размещением из n элементов по k называется любое упорядоченное множество из k элементов, состоящее из элементов заданного n-элементного множества.

Например, из множества, содержащего три цифры , можно составить следующие размещения из двух элементов без повторений:

(1; 5), (1; 7), (5; 7), (5; 1), (7; 1), (7; 5).

Количество размещений из n элементов по k обозначается Уравнения на комбинаторику с решением(читается: «А из n по k», A — первая буква французского слова arrangement, что означает «размещение, приведение в порядок»). Как видим,Уравнения на комбинаторику с решением

Выясним, сколько всего можно составить размещений из n элементов по k без повторений. Составление размещения представим себе как последовательное заполнение k мест, которые будем изображать в виде клеточек (рис. 21.1). На первое место можем выбрать один из n элементов данного множества (то есть элемент для первой клеточки можно выбрать n способами).

Если элементы нельзя повторять, то на второе место можно выбрать только один элемент из оставшихся, то есть из n – 1 элементов. Теперь уже два элемента использованы и на третье место можно выбрать только один из n – 2 элементов и т. д. На k-е место можно выбрать только один из n – (k –1) = n – k +1 элементов (см. рис. 21.1).

Уравнения на комбинаторику с решением

Поскольку требуется выбрать элементы и на первое место, и на второе, . и на k-е, то используем правило произведения и получим следующую формулу числа размещений из n элементов по k:

Уравнения на комбинаторику с решением

Например, Уравнения на комбинаторику с решением(что совпадает с соответствующим значением, полученным выше). Аналогично можно обосновать формулу для нахождения числа размещений с повторениями. При решении простейших комбинаторных задач важно правильно выбрать формулу, по которой будут проводиться вычисления. Для этого нужно выяснить следующее:

  1. Учитывается ли порядок следования элементов в соединении?
  2. Все ли заданные элементы входят в полученное соединение?

Если, например, порядок следования элементов учитывается и из n данных элементов в соединении используется только k элементов, то по определению это — размещение из n элементов по k.

После определения вида соединения следует также выяснить, могут ли элементы в соединении повторяться, то есть выяснить, какую формулу необходимо использовать — для количества соединений без повторений или с повторениями.

Примеры решения задач:

Пример:

На соревнования по легкой атлетике приехала команда из 12 спортсменок. Сколькими способами тренер может определить, кто из них побежит в эстафете 4 × 100 м на первом, втором, третьем и четвертом этапах?

Решение:

Количество способов выбрать из 12 спортсменок четырех для участия в эстафете равно количеству размещений из 12 элементов по 4 (без повторений), то есть Уравнения на комбинаторику с решением

Для выбора формулы выясняем ответы на вопросы, приведенные выше. Поскольку для спортсменок важно, в каком порядке они будут бежать, то порядок при выборе элементов учитывается. В полученное соединение входят не все 12 заданных элементов. Следовательно, соответствующее соединение — размещение из 12 элементов по 4 (без повторений, поскольку каждая спортсменка может бежать только на одном этапе эстафеты).

Пример:

Найдите количество трехзначных чисел, которые можно составить из цифр 1, 2, 3, 4, 5, 6, 7, если цифры в числе не повторяются.

Решение:

Количество трехзначных чисел, которые можно составить из семи цифр 1, 2, 3, 4, 5, 6, 7, равно числу размещений из 7 элементов по 3, то естьУравнения на комбинаторику с решением

Для выбора формулы выясняем, что для чисел, которые мы будем составлять, порядок следования цифр учитывается и не все элементы выбираются (только 3 из заданных семи). Следовательно, соответствующее соединение — размещение из 7 элементов по 3 (без повторений).

Пример:

Найдите количество трехзначных чисел, которые можно составить из цифр 1, 2, 3, 4, 5, 6, 0, если цифры в числе не повторяются.

Выбор формулы проводится таким же образом, как и в задаче 2. Следует учесть, что если число, составленное из трех цифр, начинается цифрой 0, то оно не считается трехзначным. Следовательно, для ответа на вопрос задачи можно сначала из заданных 7 цифр записать все числа, состоящие из 3 цифр (см. задачу 2). Затем из количества полученных чисел вычесть количество чисел, составленных из трех цифр, но начинающихся цифрой 0. В последнем случае мы фактически будем из всех цифр без нуля (их 6) составлять двузначные числа. Тогда их количество равно числу размещений из 6 элементов по 2 (см. решение).

Можно выполнить также непосредственное вычисление, последовательно заполняя три места в трехзначном числе и используя правило произведения. В этом случае для наглядности удобно изображать соответствующие разряды в трехзначном числе в виде клеточек, например так:

Уравнения на комбинаторику с решением

Решение:

Количество трехзначных чисел, которые можно составить из семи цифр (среди которых нет цифры 0), если цифры в числе не повторяются, равно числу размещений из 7 элементов по 3, то есть Уравнения на комбинаторику с решением

Но среди данных цифр есть цифра 0, с которой не может начинаться трехзначное число. Поэтому из размещений из 7 элементов по 3 необходимо исключить те размещения, в которых первым элементом является цифра 0. Их количество равно числу размещений из 6 элементов по 2, то есть Уравнения на комбинаторику с решениемСледовательно, искомое количество трехзначных чисел равно Уравнения на комбинаторику с решением

Пример:

Решите уравнениеУравнения на комбинаторику с решением

Решение:

ОДЗ: x ∈ N, Уравнения на комбинаторику с решением. Тогда получаем: Уравнения на комбинаторику с решением

На ОДЗ это уравнение равносильно уравнениям:

Тогда x = 0 или x = 5. В ОДЗ входит только x = 5.

Уравнения, в запись которых входят выражения, обозначающие количество соответствующих соединений из x элементов, считаются определенными только при натуральных значениях переменной x. Чтобы выражение Уравнения на комбинаторику с решениемимело смысл, следует выбирать натуральные значения Уравнения на комбинаторику с решением(в этом случае Уравнения на комбинаторику с решениемтакже существует и, конечно, Ax 2 ≠ 0). Для преобразования уравнения используем формулы:Уравнения на комбинаторику с решением

Объяснение и обоснование:

Перестановкой из n элементов называется любое упорядоченное множество из n заданных элементов.

Напомним, что упорядоченное множество — это такое множество, для которого указано, какой элемент находится на первом месте, какой на втором, . какой на n-м.

Например, переставляя цифры в числе 236 (в котором множество цифр уже упорядоченное), можно составить такие перестановки без повторений: (2; 3; 6), (2; 6; 3), (3; 2; 6), (3; 6; 2), (6; 2; 3), (6; 3; 2) — всего 6 перестановок* .

Количество перестановок без повторений из n элементов обозначается Уравнения на комбинаторику с решением(P — первая буква французского слова permutation — перестановка). Как видим, Уравнения на комбинаторику с решением= 6.

Фактически перестановки без повторений из n элементов являются размещениями из n элементов по n без повторений, поэтому Уравнения на комбинаторику с решениемПроизведение Уравнения на комбинаторику с решениемобозначается n!. Поэтому полученная формула числа перестановок без повторений из n элементов может быть записана следующим образом:

Уравнения на комбинаторику с решением

*Отметим, что каждая из перестановок определяет трехзначное число, составленное из цифр 2, 3, 6 таким образом, что цифры в числе не повторяются.

Например, Уравнения на комбинаторику с решением(что совпадает с соответствующим значением, полученным выше).

С помощью факториалов формулу для числа размещений без повторений

Уравнения на комбинаторику с решением(1)

запишем в другом виде. Для этого умножим и разделим выражение в формуле (1) на произведение Уравнения на комбинаторику с решениемтогда

Уравнения на комбинаторику с решением

Следовательно, формула числа размещений без повторений из n элементов по k может быть записана так:

Уравнения на комбинаторику с решением(2)

Для того чтобы этой формулой можно было пользоваться при всех значениях k, в частности при k = n – 1 и k = n, договорились считать, что

Например, по формуле (2) Уравнения на комбинаторику с решением

Обратим внимание, что в тех случаях, когда значение n! оказывается очень большим, ответы оставляют записанными с помощью факториалов. Например,Уравнения на комбинаторику с решением

Примеры решения задач:

Для выбора формулы при решении простейших комбинаторных задач достаточно выяснить следующее:

  1. Учитывается ли порядок следования элементов в соединении?
  2. Все ли заданные элементы входят в полученное соединение?

Если, например, порядок следования элементов учитывается и все n заданных элементов используются в соединении, то по определению это перестановки из n элементов.

Пример:

Найдите, сколькими способами можно восемь учащихся построить в колонну по одному.

Решение:

Количество способов равно числу перестановок из 8 элементов, то есть Уравнения на комбинаторику с решением

Для выбора соответствующей формулы выясняем ответы на вопросы, приведенные выше. Поскольку порядок следования элементов учитывается и все 8 заданных элементов выбираются, то искомые соединения — это перестановки из 8 элементов без повторений. Их количество можно вычислить по формуле Уравнения на комбинаторику с решением

Пример:

Найдите количество различных четырехзначных чисел, которые можно составить из цифр 0, 3, 7, 9 (цифры в числе не повторяются).

Решение:

Из четырех цифр 0, 3, 7, 9, не повторяя заданные цифры, можно получить Уравнения на комбинаторику с решениемперестановок. Перестановки, начинающиеся с цифры 0, не являются записью четырехзначного числа — их количество Уравнения на комбинаторику с решением. Тогда искомое количество четырехзначных чисел равноУравнения на комбинаторику с решением

Поскольку порядок следования элементов учитывается и для получения четырехзначного числа надо использовать все элементы, то искомые соединения — это перестановки из 4 элементов. Их количество — Уравнения на комбинаторику с решением. При этом необходимо учесть, что в четырехзначном числе на первом месте не может стоять цифра 0. Таких чисел будет столько, сколько раз мы сможем выполнить перестановки из 3 оставшихся цифр, то есть Уравнения на комбинаторику с решением

Пример:

Имеется десять книг, из которых четыре — учебники. Сколькими способами можно поставить эти книги на полку так, чтобы все учебники стояли рядом?

Решение:

Сначала будем рассматривать учебники как одну книгу. Тогда на полке надо расставить не 10, а 7 книг. Это можно сделать Уравнения на комбинаторику с решениемспособами. В каждом из полученных наборов книг можно выполнить еще Уравнения на комбинаторику с решениемперестановок учебников. По правилу умножения искомое количество способов равноУравнения на комбинаторику с решением

Задачу можно решать в два этапа. На первом будем условно считать все учебники одной книгой.

Тогда получим 7 книг (6 не учебников + 1 условная книга — учебник). Порядок следования элементов учитывается и используются все элементы (поставить на полку необходимо все книги). Следовательно, соответствующие соединения — это перестановки из 7 элементов. Их количество — Уравнения на комбинаторику с решением.

На втором этапе решения будем переставлять между собой только учебники. Это можно сделать Уравнения на комбинаторику с решениемспособами. Поскольку нам надо переставить и учебники, и другие книги, то используем правило произведения.

Объяснение и обоснование:

1. Сочетания без повторений:

Сочетанием без повторений из n элементов по k называется любое k-элементное подмножество заданного n-элементного множества.

Например, из множества можно составить следующие сочетания без повторений из трех элементов: , , , .

Количество сочетаний без повторений из n элементов по k элементов обозначается символом Уравнения на комбинаторику с решением(читается: «число сочетаний из п по k» или «це из п по k», С — первая буква французского слова combinaison — сочетание). Как видим, Уравнения на комбинаторику с решением

Выясним, сколько всего можно составить сочетаний без повторений из n элементов по k. Для этого используем известные нам формулы числа размещений и перестановок. Составление размещения без повторений из n элементов по k проведем в два этапа. Сначала выберем k разных элементов из заданного n-элементного множества, не учитывая порядок выбора этих элементов (то есть выберем kэлементное подмножество из n-элементного множества — сочетание без повторений из n-элементов по k). По нашему обозначению это можно сделать Уравнения на комбинаторику с решениемспособами. После этого полученное множество из k разных элементов упорядочим. Его можно упорядочить Уравнения на комбинаторику с решениемспособами. Получим размещения без повторений из n элементов по k. Следовательно, количество размещений без повторений из n элементов по k в k! раз больше числа сочетаний без повторений из n элементов по k, то естьУравнения на комбинаторику с решениемОтсюда Уравнения на комбинаторику с решениемУчитывая, что по формуле (2) Уравнения на комбинаторику с решением, получаем:

Уравнения на комбинаторику с решением(3)

Например, Уравнения на комбинаторику с решениемчто совпадает со значением, полученным выше.

Используя формулу (3), можно легко обосновать свойство 1 числа сочетаний без повторений, приведенное в табл. 28.

1) Поскольку Уравнения на комбинаторику с решениемто

Уравнения на комбинаторику с решением(4)

Для того чтобы формулу (4) можно было использовать и при k = n, договорились считать, что Уравнения на комбинаторику с решениемТогдаУравнения на комбинаторику с решением

Заметим, что формулу (4) можно получить без вычислений с помощью достаточно простых комбинаторных рассуждений.

Когда мы выбираем k предметов из n, то n – k предметов мы оставляем. Если же, напротив, выбранные предметы оставим, а другие n – k -выберем, то получим способ выбора n – k предметов из n. Мы получили взаимно-однозначное соответствие способов выбора k и n – k предметов из n. Значит, количество одних и других способов одинаково. Но количество одних — Уравнения на комбинаторику с решением, а других Уравнения на комбинаторику с решением, поэтому Уравнения на комбинаторику с решением.

Если в формуле (3) сократить числитель и знаменатель на (n – k)!, то получим формулу, по которой удобно вычислять Уравнения на комбинаторику с решениемпри малых значениях k:

Уравнения на комбинаторику с решением(5)

Например,Уравнения на комбинаторику с решением

2. Вычисление числа сочетаний без повторений с помощью треугольника Паскаля:

Для вычисления числа сочетаний без повторений можно применять формулу (3): Уравнения на комбинаторику с решением, а можно последовательно вычислять соответствующие значения, пользуясь следующим свойством:

Уравнения на комбинаторику с решением(6)

Для обоснования равенства (6) можно записать суммуУравнения на комбинаторику с решением, используя формулу (3), и после приведения полученных дробей к общему знаменателю получить формулу для правой части равенства (6) (проделайте это самостоятельно). Также формулу (6) можно получить без вычислений с помощью комбинаторных рассуждений.

Уравнения на комбинаторику с решением— это количество способов выбрать k +1 предмет из n + 1. Подсчитаем это количество, зафиксировав один предмет (назовем его «фиксированным»). Если мы не берем фиксированный предмет, то нам нужно выбрать k +1 предмет из n тех, что остались, а если мы его берем, то нужно выбрать из n тех, что остались, еще k предметов. Первое можно сделать Уравнения на комбинаторику с решениемспособами, второеУравнения на комбинаторику с решениемспособами. Всего как раз Уравнения на комбинаторику с решениемспособов, следовательно,

Уравнения на комбинаторику с решением

Это равенство позволяет последовательно вычислять значения Уравнения на комбинаторику с решениемс помощью специальной таблицы, которая называется треугольником Паскаля. Если считать, что Уравнения на комбинаторику с решением, то он будет иметь вид, представленный в табл. 29.

Уравнения на комбинаторику с решением

Каждая строка этой таблицы начинается с единицы и заканчивается единицейУравнения на комбинаторику с решением

Если какая-либо строка уже заполнена, например третья, то в четвертой строке надо записать на первом месте единицу. На втором месте запишем число, равное сумме двух чисел третьей строки, стоящих над ним левее и правее (поскольку по формуле (6) Уравнения на комбинаторику с решениемНа третьем месте запишем число, равное сумме двух следующих чисел третьей строки, стоящих над ним левее и правее Уравнения на комбинаторику с решением, и т. д. (а на последнем месте снова запишем единицу).

Примеры решения задач:

Обратим внимание, что, как и раньше, для выбора формулы при решении простейших комбинаторных задач достаточно ответить на вопросы:

  1. Учитывается ли порядок следования элементов в соединении?
  2. Все ли заданные элементы входят в полученное соединение?

Чтобы выяснить, является ли заданное соединение сочетанием, достаточно ответить только на первый вопрос (см. схему в табл. 28). Если порядок следования элементов не учитывается, то по определению это сочетание из n элементов по k элементов.

Пример:

Из 12 членов туристической группы надо выбрать трех дежурных. Сколькими способами можно сделать этот выбор?

Решение:

Количество способов выбрать из 12 туристов трех дежурных равно количеству сочетаний из 12 элементов по 3 (без повторений), то естьУравнения на комбинаторику с решением

Для выбора соответствующей формулы выясняем ответы на вопросы, приведенные выше. Поскольку порядок следования элементов не учитывается (для дежурных неважно, в каком порядке их выберут), то соответствующее соединение является сочетанием из 12 элементов по 3 (без повторений). Для вычисления можно использовать формулы (3) или (5), в данном случае применяем формулу (3):Уравнения на комбинаторику с решением

Пример:

Из вазы с фруктами, в которой лежат 10 разных яблок и 5 разных груш, требуется выбрать 2 яблока и 3 груши. Сколькими способами можно сделать такой выбор?

Решение:

Выбрать 2 яблока из 10 можно Уравнения на комбинаторику с решениемспособами. При каждом выборе яблок груши можно выбрать Уравнения на комбинаторику с решениемспособами. Тогда по правилу произведения выбор требуемых фруктов можно выполнить Уравнения на комбинаторику с решениемспособами. ПолучаемУравнения на комбинаторику с решением

Сначала отдельно выберем 2 яблока из 10 и 3 груши из 5.

Поскольку при выборе яблок или груш порядок следования элементов не учитывается, то соответствующие соединения — сочетания без повторений.

Учитывая, что требуется выбрать 2 яблока и 3 груши, используем правило произведения и перемножим полученные возможности выбора яблок Уравнения на комбинаторику с решениеми груш Уравнения на комбинаторику с решением

Бином Ньютона:

Уравнения на комбинаторику с решением

Поскольку Уравнения на комбинаторику с решением(при x ≠ 0 и a ≠ 0), то формулу бинома Ньютона можно записать еще и так:

Уравнения на комбинаторику с решением

Общий член разложения степени бинома имеет вид

Уравнения на комбинаторику с решением(где Уравнения на комбинаторику с решением). Коэффициенты Уравнения на комбинаторику с решениемназывают биномиальными коэффициентaми.

Свойства биномиальных коэффициентов:

  1. Число биномиальных коэффициентов (а следовательно, и число слагаемых) в разложении n-й степени бинома равно n + 1.
  2. Коэффициенты членов, равноудаленных от начала и конца разложения, равны между собой (поскольку Уравнения на комбинаторику с решением)
  3. Сумма всех биномиальных коэффициентов равна Уравнения на комбинаторику с решениемУравнения на комбинаторику с решением
  4. Сумма биномиальных коэффициентов, стоящих на четных местах, равна сумме биномиальных коэффициентов, стоящих на нечетных местах.
  5. Для вычисления биномиальных коэффициентов можно воспользоваться треугольником Паскаля, в котором вычисления коэффициентов основываются на формуле Уравнения на комбинаторику с решением

Уравнения на комбинаторику с решением

Объяснение и обоснование:

Бином Ньютона:

Двучлен вида a + x также называют биномом. Из курса алгебры известно, что:

Уравнения на комбинаторику с решением

Можно заметить, что коэффициенты разложения степени бинома Уравнения на комбинаторику с решениемпри n = 1, 2, 3 совпадают с числами в соответствующей строке треугольника Паскаля. Оказывается, что это свойство выполняется для любого натурального n, то есть справедлива формула

Уравнения на комбинаторику с решением(7)

Формулу (7) называют биномом Ньютона. Правая часть этого равенства называется разложением степени биномаУравнения на комбинаторику с решением, а числа Уравнения на комбинаторику с решением(при k = 0, 1, 2, . n) называют биномиальными коэффициентами.

Общий член разложения степени бинома имеет вид

Уравнения на комбинаторику с решением

Обосновать формулу (7) можно, например, с помощью метода математической индукции. (Проведите такое обоснование самостоятельно.)

Приведем также комбинаторные рассуждения для обоснования формулы бинома Ньютона.

По определению степени с натуральным показателем Уравнения на комбинаторику с решением Уравнения на комбинаторику с решением(всего n скобок). Раскрывая скобки, получаем в каждом слагаемом произведение n букв, каждая из которых — а или х. Если, например, в каком-либо слагаемом количество букв x равно k, то количество букв а в нем — n – k, то есть каждое слагаемое имеет вид Уравнения на комбинаторику с решениемпри некотором k от 0 до n. Покажем, что для каждого такого k число слагаемых anУравнения на комбинаторику с решениемравно Уравнения на комбинаторику с решением, откуда после приведения подобных членов и получаем формулу бинома. Произведение Уравнения на комбинаторику с решениемполучаем, взяв букву x из k скобок и букву а из n – k тех скобок, которые остались. Разные такие слагаемые получим путем разного выбора первых k скобок, а k скобок из n можно выбрать именно Уравнения на комбинаторику с решениемспособами. Следовательно, общий член разложения бинома Уравнения на комбинаторику с решениемдействительно имеет вид Уравнения на комбинаторику с решениемгде k = 0, 1, 2, . n.

Именно из-за бинома Ньютона числа Уравнения на комбинаторику с решениемчасто называют биномиальными коэффициентами.

Записывая степень двучлена по формуле бинома Ньютона для небольших значений n, биномиальные коэффициенты можно вычислять с помощью треугольника Паскаля (см. табл. 30).

Например, Уравнения на комбинаторику с решением

Так как Уравнения на комбинаторику с решением, формулу бинома Ньютона можно записать в виде:

Уравнения на комбинаторику с решением(8)

Если в формуле бинома Ньютона (8) заменить x на (–x), то получим формулу возведения в степень разности a – x:

Уравнения на комбинаторику с решением

Например, Уравнения на комбинаторику с решением(знаки членов разложения чередуются!).

Свойства биномиальных коэффициентов:

  1. Число биномиальных коэффициентов (а следовательно, и число слагаемых) в разложении n-й степени бинома равно n + 1, поскольку разложение содержит все степени x от 0 до n (и других слагаемых не содержит).
  2. Коэффициенты членов, равноудаленных от начала и конца разложения, равны между собой, поскольку Уравнения на комбинаторику с решением
  3. Сумма всех биномиальных коэффициентов равнаУравнения на комбинаторику с решением

Для обоснования полагаем в равенстве (7) значения a = x = 1 и получаем:

Уравнения на комбинаторику с решением

Например, Уравнения на комбинаторику с решением

4. Сумма биномиальных коэффициентов, стоящих на четных местах, равна сумме биномиальных коэффициентов, стоящих на нечетных местах.

Для обоснования возьмем в равенстве (7) значения a = 1, x = –1:

Уравнения на комбинаторику с решением

Тогда Уравнения на комбинаторику с решением

Примеры решения задач:

Пример:

По формуле бинома Ньютона найдите разложение степениУравнения на комбинаторику с решением.

Для нахождения коэффициентов разложения можно использовать треугольник Паскаля (табл. 30) или вычислять их по общей формуле. По треугольнику Паскаля коэффициенты равны: 1, 6, 15, 20, 15, 6, 1. Учитывая, что при возведении разности в степень знаки членов разложения чередуются, получаем:

Уравнения на комбинаторику с решением

Для упрощения записи ответа можно избавиться от иррациональности в знаменателях полученных выражений (см. решение) или сначала учесть, что ОДЗ данного выражения: x > 0. Тогда Уравнения на комбинаторику с решениемто есть данное выражение можно записать так: Уравнения на комбинаторику с решениеми возвести в степень последнее выражение.

Решение:

Уравнения на комбинаторику с решением

Пример:

В разложении степени Уравнения на комбинаторику с решениемнайдите член, содержащий Уравнения на комбинаторику с решением

Решение:

Уравнения на комбинаторику с решением.

Общий член разложения: Уравнения на комбинаторику с решением

По условию член разложения должен содержать Уравнения на комбинаторику с решением, следовательно, Уравнения на комбинаторику с решениемОтсюда k = 6.

Тогда член разложения, содержащий Уравнения на комбинаторику с решением, равен

Уравнения на комбинаторику с решением

На ОДЗ (b > 0) каждое слагаемое в данном двучлене можно записать как степень с дробным показателем. Это позволит проще записать общий член разложения степени Уравнения на комбинаторику с решением

Уравнения на комбинаторику с решением

(где k = 0, 1, 2, . n), выяснить, какой из членов разложения содержит Уравнения на комбинаторику с решениеми записать его. Чтобы упростить запись общего члена разложения, запишем:

Уравнения на комбинаторику с решением

Всё о комбинаторике

Пусть имеется несколько множеств элементов:

Уравнения на комбинаторику с решением

Вопрос: сколькими способами можно составить новое множество Уравнения на комбинаторику с решениемвзяв из каждого исходного множества по одному элементу? Ответ на этот вопрос дают следующие рассуждения.

Элемент Уравнения на комбинаторику с решениемиз первого множества можно выбрать Уравнения на комбинаторику с решениемспособами, элемент Уравнения на комбинаторику с решениемиз второго – s способами, элемент с можно выбрать Уравнения на комбинаторику с решениемспособами и т. д. Пару элементов Уравнения на комбинаторику с решениемможно составить Уравнения на комбинаторику с решениемs способами. Это следует из табл. 1.1, в которой перечислены все способы такого выбора.

Уравнения на комбинаторику с решением

Способы выбора трех элементов аbc перечислены в табл. 1.2.

Уравнения на комбинаторику с решением

В этой таблице Уравнения на комбинаторику с решениемстрок и Уравнения на комбинаторику с решениемs столбцов. Поэтому искомое число способов выбора трех элементов аbc равно Уравнения на комбинаторику с решениемs Уравнения на комбинаторику с решением. Продолжая рассуждать подобным образом, получим следующее утверждение.

Основной комбинаторный принцип. Если некоторый первый выбор можно сделать Уравнения на комбинаторику с решением способами, для каждого первого выбора некоторый второй можно сделать s способами, для каждой пары первых двух – третий выбор можно сделать Уравнения на комбинаторику с решением способами и т.д., то число способов для последовательности таких выборов равно Уравнения на комбинаторику с решениемs Уравнения на комбинаторику с решением.

Комбинаторные формулы в прикладных задачах теории вероятностей обычно связывают с выбором Уравнения на комбинаторику с решениемэлементов («выборкой объема Уравнения на комбинаторику с решением») из совокупности, состоящей из Уравнения на комбинаторику с решениемэлементов (элементов «генеральной совокупности»). Различают два способа выбора:

  • а) повторный выбор, при котором выбранный элемент возвращается в генеральную совокупность и может быть выбран вновь;
  • б) бесповторный выбор, при котором выбранный элемент в совокупность не возвращается и выборка не содержит повторяющихся элементов.

При повторном выборе каждый по порядку элемент может быть выбран Уравнения на комбинаторику с решениемспособами. Согласно комбинаторному принципу, такую выборку можно сделать Уравнения на комбинаторику с решениемспособами. Например, повторную выборку объема 2 из трех элементов Уравнения на комбинаторику с решениемможно сделать 3 2 =9 способами: Уравнения на комбинаторику с решениемУравнения на комбинаторику с решением

В случае бесповторной выборки первый элемент можно выбрать Уравнения на комбинаторику с решениемспособами, для второго остается Уравнения на комбинаторику с решениемвозможность выбора, третий элемент можно выбрать Уравнения на комбинаторику с решениемспособами и т.д. Элемент выборки с номером Уравнения на комбинаторику с решениемможно выбрать Уравнения на комбинаторику с решениемспособом. Согласно комбинаторному принципу, общее число бесповторных выборок объема Уравнения на комбинаторику с решениемравно

Уравнения на комбинаторику с решением

Число Уравнения на комбинаторику с решениемназывают числом размещений из Уравнения на комбинаторику с решениемэлементов по Уравнения на комбинаторику с решением.

Например, существует Уравнения на комбинаторику с решениемразмещений из трех элементов Уравнения на комбинаторику с решениемпо два: Уравнения на комбинаторику с решениемОтметим, что и в первом случае и во втором выборки отличаются либо составом элементов, либо порядком выбора элементов.

Выделим особо случай, когда один за другим выбраны все Уравнения на комбинаторику с решениемэлементов. В этом случае выборки имеют один и тот же состав (все Уравнения на комбинаторику с решениемэлементов) и отличаются только порядком выбора элементов. Поэтому число

Уравнения на комбинаторику с решением

называют числом перестановок из Уравнения на комбинаторику с решениемэлементов.

Например, пять человек могут встать в очередь Уравнения на комбинаторику с решениемспособами. Три элемента Уравнения на комбинаторику с решениемможно переставить Уравнения на комбинаторику с решениемспособами: Уравнения на комбинаторику с решением

Подсчитаем количество бесповторных выборок объема Уравнения на комбинаторику с решением, которые отличаются друг от друга только составом элементов. Пусть X — число таких выборок. Для каждого набора из Уравнения на комбинаторику с решениемэлементов можно выбрать порядок их расположения Уравнения на комбинаторику с решениемспособами. Тогда Уравнения на комбинаторику с решениемравно числу способов выбрать Уравнения на комбинаторику с решениемразличных элементов и выбрать порядок их расположения, т.е. равно числу размещений из Уравнения на комбинаторику с решениемэлементов по Уравнения на комбинаторику с решением:

Уравнения на комбинаторику с решением

Это число называют числом сочетаний из Уравнения на комбинаторику с решениемэлементов по Уравнения на комбинаторику с решением и обозначают через Уравнения на комбинаторику с решениемЕсли в формуле (1.2) умножить числитель и знаменатель на Уравнения на комбинаторику с решением, то

Уравнения на комбинаторику с решением

Например, сочетаний из четырех элементов Уравнения на комбинаторику с решениемпо два существует Уравнения на комбинаторику с решением. Это Уравнения на комбинаторику с решением

Так как из Уравнения на комбинаторику с решением элементов выбрать Уравнения на комбинаторику с решением элементов можно единственным образом, то Уравнения на комбинаторику с решениемоткуда следует, что Уравнения на комбинаторику с решением

Величины Уравнения на комбинаторику с решениемназывают биномиальными коэффициентами. Название связано с формулой бинома Ньютона

Уравнения на комбинаторику с решением

Из формулы (1.3) следует, что

Уравнения на комбинаторику с решением

Биномиальные коэффициенты образуют так называемый треугольник Паскаля, который имеет вид:

Уравнения на комбинаторику с решением

В Уравнения на комбинаторику с решением-й строке треугольника Паскаля располагаются коэффициенты, соответствующие представлению Уравнения на комбинаторику с решениемпо формуле (1.3). Треугольником удобно пользоваться для нахождения значений Уравнения на комбинаторику с решением. Это значение находится на пересечении Уравнения на комбинаторику с решением-й строки и Уравнения на комбинаторику с решением-го наклонного ряда. Например, Уравнения на комбинаторику с решением

Биномиальные коэффициенты обладают свойством симметрии:

Уравнения на комбинаторику с решением

Это наглядно демонстрирует треугольник Паскаля. Равенство (1.4) подтверждает тот очевидный факт, что выбор Уравнения на комбинаторику с решением элементов из n равносилен выбору тех Уравнения на комбинаторику с решениемУравнения на комбинаторику с решением элементов из Уравнения на комбинаторику с решением, которые следует удалить, чтобы остались Уравнения на комбинаторику с решением элементов.

При повторном выборе из Уравнения на комбинаторику с решением элементов число выборок объема Уравнения на комбинаторику с решением, которые отличаются только составом равно Уравнения на комбинаторику с решениемЕще раз подчеркнем, что речь идет о выборках, которые отличаются хотя бы одним элементом, а порядок выбора этих элементов во внимание не принимается. Число таких выборок можно подсчитать следующим образом. Между элементами Уравнения на комбинаторику с решениемпоставим разграничительные знаки, например, нули: Уравнения на комбинаторику с решениемТаких знаков (нулей) понадобится Уравнения на комбинаторику с решением. На месте каждого элемента поставим столько единиц, сколько раз предполагается выбрать этот элемент. Например, комбинация Уравнения на комбинаторику с решениемозначает, что элемент Уравнения на комбинаторику с решениемвыбран четыре раза, элемент Уравнения на комбинаторику с решениемвыбран один раз, элемент Уравнения на комбинаторику с решениемне выбран, . элемент Уравнения на комбинаторику с решениемвыбран два раза. Заметим, что в такой записи число единиц равно объему выборки Уравнения на комбинаторику с решением. Для перебора всех возможных комбинаций нужно из Уравнения на комбинаторику с решениеммест выбрать Уравнения на комбинаторику с решениемместо и поставить на них нули, а на остальных местах разместить единицы. Это можно сделать способами.

Уравнения на комбинаторику с решением

Совокупность из Уравнения на комбинаторику с решением элементов разделить на Уравнения на комбинаторику с решениемгрупп по Уравнения на комбинаторику с решениемэлементов соответственно Уравнения на комбинаторику с решениемможно Уравнения на комбинаторику с решениемспособами. Порядок элементов внутри каждой из этих Уравнения на комбинаторику с решениемгрупп не имеет значения.

Пусть Уравнения на комбинаторику с решением– множества, число элементов в каждом из которых равно соответственно Уравнения на комбинаторику с решениемСоставить множество B из Уравнения на комбинаторику с решениемэлементов множества А1, Уравнения на комбинаторику с решениемэлементов множества А2, …, Уравнения на комбинаторику с решениемэлементов множества Аk, можно, согласно основному комбинаторному принципу, способами.

Уравнения на комбинаторику с решением

Для безошибочного выбора комбинаторной формулы достаточно последовательно ответить на вопросы в следующей схеме:

Уравнения на комбинаторику с решением

Например, число словарей, необходимых для непосредственного перевода с одного на другой, для пяти языков определяется из следующих рассуждений. Для составления словаря выбираем из пяти языков (Уравнения на комбинаторику с решением= 5) любые два (Уравнения на комбинаторику с решением=2). Выбор бесповторный, причем при выборе важен и состав выбора и порядок выбора. Поэтому искомое число словарей равно Уравнения на комбинаторику с решением

Комбинаторные задачи с решением

Комбинаторика — раздел математики, занимающийся вопросом выбора и расположения элементов некоторого конечного множества в соответствии с заданными условиями.

Рассмотрим примеры задач комбинаторики.

Пример №1

Сколькими способами можно выбрать путь из начала координат 0(0,0) в точку В(6,4), если каждый шаг равен единице, но его можно совершать только вправо или вверх? Сколько таких путей проходит через точку А(2,3)?

Решение. Весь путь занимает 10 шагов (четыре вверх и шесть вправо). Для планирования пути следует решить, какие именно по счету четыре шага следует сделать вверх, а остальные шесть — вправо. Выбор бесповторный и нас интересует только состав выбора. Поэтому в описанных условиях всего путей из точки О в точку В будет Уравнения на комбинаторику с решением

Рассуждая подобным образом легко видеть, что путей из точки О в точку А существует Уравнения на комбинаторику с решениема путь из точки А в точку В можно выбрать Уравнения на комбинаторику с решениемспособами. По комбинаторному принципу всего путей через точку А существует 10 • 5 = 50.

Пример №2

Сколькими способами можно выбрать путь из начала координат 0(0,0) в точку Уравнения на комбинаторику с решениемесли каждый шаг равен 1, но его можно совершать только вправо или вверх? Сколько таких путей проходит через точку Уравнения на комбинаторику с решением(См. пример 1.1 и исходные данные.)

Исходные данные к задаче 1.1.

Уравнения на комбинаторику с решением

Пример №3

В городе с идеальной прямоугольной планировкой (сеть улиц в этом городе изображена на рис. 1.1) из пункта А выходят Уравнения на комбинаторику с решениемчеловек. Половина из них идет по направлению Уравнения на комбинаторику с решениемполовина — по направлению Уравнения на комбинаторику с решениемДойдя до первого перекрестка, каждая группа разделяется так, что половина ее идет по направлению Уравнения на комбинаторику с решениемполовина — по направлению Уравнения на комбинаторику с решениемТакое же разделение происходит на каждом перекрестке. Требуется перечислить перекрестки, на которых окажутся люди после прохождения N улиц (отрезков на рис. 1.1), и сколько людей окажется на каждом из этих перекрестков.

Уравнения на комбинаторику с решением

Решение. Каждый человек пройдет N улиц и окажется на одном из перекрестков Уравнения на комбинаторику с решениемКоординаты перекрестков указаны в предположении, что точка А служит началом координат.

На каждом перекрестке для каждого человека производится выбор из двух возможностей: идти в направлении Уравнения на комбинаторику с решениемили в направлении Уравнения на комбинаторику с решениемПоэтому всего возможных путей будет Уравнения на комбинаторику с решением. Из этого следует, что каждый путь пройдет только один человек.

В пункте Уравнения на комбинаторику с решениемокажется столько человек, сколько различных путей ведет в этот пункт из точки А . Чтобы попасть в пункт Уравнения на комбинаторику с решениемнеобходимо из N улиц выбрать бесповторным способом к улиц в направлении Уравнения на комбинаторику с решением. Это можно сделать Уравнения на комбинаторику с решениемспособами.

Ответ. Уравнения на комбинаторику с решением

Пример №4

Сколькими способами можно Уравнения на комбинаторику с решением одинаковых предметов распределить между Уравнения на комбинаторику с решениемлицами так, чтобы каждый получил не менее одного предмета?

Решение. Поставим эти предметы в ряд. Между ними будет Уравнения на комбинаторику с решениемпромежуток. В любые Уравнения на комбинаторику с решениемиз этих промежутков поставим разделяющие перегородки. Тогда все предметы разделятся на Уравнения на комбинаторику с решениемнепустых частей. Первую часть передадим первому лицу, вторую — второму и т.д. Выбрать же Уравнения на комбинаторику с решениемпромежуток из Уравнения на комбинаторику с решениемпромежутка можно Уравнения на комбинаторику с решениемспособами. Заметим, что вообще Уравнения на комбинаторику с решением предметов распределить между Уравнения на комбинаторику с решениемлицами можно Уравнения на комбинаторику с решениемспособами.

Ответ. Уравнения на комбинаторику с решением

Пример 1.4.

Сколькими способами можно распределить 6 яблок, 8 груш и 10 слив между тремя детьми? Сколькими способами это можно сделать так, чтобы каждый ребенок получил по меньшей мере одно яблоко, одну сливу и одну грушу?

Решение. Яблоки в соответствии с формулой (1.5) можно распределить Уравнения на комбинаторику с решениемспособами, груши — Уравнения на комбинаторику с решением, а сливы Уравнения на комбинаторику с решениемспособами. По комбинаторному принципу всего способов Уравнения на комбинаторику с решениемЕсли необходимо, чтобы каждый ребенок получил по меньшей мере одно яблоко, одну грушу и одну сливу, то в соответствии с формулой предыдущего примера имеем Уравнения на комбинаторику с решениемспособов.

Пример №5

Сколько цифр в первой тысяче не содержат в своей записи цифры 5?

Решение. Для записи любой из цифр 000, 001, 002, . 999 необходимо трижды выбрать повторным способом одну из десяти цифр, поэтому и получается всего Уравнения на комбинаторику с решениемчисел. Если цифру 5 исключить, то выбор можно производить только из девяти цифр: 0, 1,2, 3, 4, 6, 7, 8, 9. Поэтому всего получится Уравнения на комбинаторику с решениемчисел в первой тысяче, в записи которых нет цифры 5.

Пример №6

Сколько шестизначных чисел содержат в записи ровно три различных цифры?

Решение. Заметим, что всего шестизначных чисел имеется Уравнения на комбинаторику с решением, так как первая цифра может быть любой (исключая нуль), а остальные пять могут быть выбраны Уравнения на комбинаторику с решениемспособами.

Выбрать три ненулевых цифры можно Уравнения на комбинаторику с решениемспособами. Из выбранных трех цифр можно составить Уравнения на комбинаторику с решениемшестизначных чисел, из двух — Уравнения на комбинаторику с решением, а из одной — Уравнения на комбинаторику с решениемшестизначное число. По формуле (1.7) получаем, что существует Уравнения на комбинаторику с решениемшестизначных чисел, в записи которых есть только три заданные цифры. Поэтому общее число шестизначных чисел, в записи которых имеются три отличные от нуля цифры, равно Уравнения на комбинаторику с решением

Учтем теперь возможность использования нуля. К нулю нужно добавить две цифры, что можно сделать Уравнения на комбинаторику с решениемспособами. Если, например, были выбраны цифры 0, 2, 5, то первой цифрой должна быть 2 или 5. К этой первой цифре в соответствии с формулой (1.7) можно добавить Уравнения на комбинаторику с решениемкомбинаций остальных пяти цифр. Тогда всего шестизначных чисел, состоящих из 0, 2, 5 будет Уравнения на комбинаторику с решениемВсего же шестизначных чисел, записанных тремя цифрами, среди которых встречается нуль, ровно Уравнения на комбинаторику с решениемВсего чисел, удовлетворяющих условиям задачи, имеется Уравнения на комбинаторику с решением

Пример №7

В саду есть цветы десяти наименований (розы, флоксы, ромашки и т. д.).

а) Сколькими способами можно составить букет из пяти цветков (не принимая во внимание совместимость растений и художественные соображения)?

б) Сколькими способами можно составить букет из пяти различных цветков?

в) Сколькими способами можно составить букет из пяти цветков так, чтобы в букете непременно было хотя бы по одному цветку двух определенных наименований

Решение. а) Если запрета на повторение цветков нет, то мы имеем дело с повторным выбором и нас интересует только состав. Поэтому по формуле (1.5) получаем Уравнения на комбинаторику с решениемспособа.

б) Если цветы должны быть разными, то способ выбора бесповторный и букет можно составить Уравнения на комбинаторику с решениемспособами.

в) Отберем по одному цветку каждого из двух названных наименований. Три остальных цветка можно выбрать из 10 возможных Уравнения на комбинаторику с решениемспособами.

Ответ. а) 2002; б) 504; в) 220.

Пример №8

Имеется Уравнения на комбинаторику с решениемяблок, Уравнения на комбинаторику с решениемгруш и Уравнения на комбинаторику с решениемперсиков. Сколькими способами можно их разложить по двум корзинам? Сколькими способами можно это сделать, если в каждой корзине должно быть хотя бы по одному фрукту всех названных видов (полагаем, что фруктов каждого наименования два или больше)?

Решение. Ясно, что яблоки можно разложить Уравнения на комбинаторику с решениемспособом (в первую корзину можно не положить яблок совсем, положить одно яблоко, два яблока, …, все яблоки). Те же рассуждения в отношении груш и персиков дают соответственно Уравнения на комбинаторику с решениемкомбинаций. По комбинаторному принципу всего будет Уравнения на комбинаторику с решениемспособов.

При ответе на второй вопрос учтем, что следует по одному яблоку сразу положить в каждую из корзин, а остальные Уравнения на комбинаторику с решениемяблока раскладывать произвольным образом (в первую корзину либо не добавляем яблок, либо добавляем одно, либо –– два, …, либо – все Уравнения на комбинаторику с решениемяблока). Все это можно сделать Уравнения на комбинаторику с решениемспособами. Те же рассуждения насчет других фруктов и комбинаторный принцип дают следующий результат: Уравнения на комбинаторику с решением

Ответ. Уравнения на комбинаторику с решением

Пример №9

Требуется найти число натуральных делителей натурального числа Уравнения на комбинаторику с решением.

Решение. Разложим Уравнения на комбинаторику с решениемна простые множители:

Уравнения на комбинаторику с решением

где Уравнения на комбинаторику с решением– различные простые числа. (Например, Уравнения на комбинаторику с решениемУравнения на комбинаторику с решением)

Заметим, что при разделении числа Уравнения на комбинаторику с решениемна любые два множителя Уравнения на комбинаторику с решениеми Уравнения на комбинаторику с решениемпростые сомножители распределятся между Уравнения на комбинаторику с решениеми Уравнения на комбинаторику с решением. Если сомножитель , Уравнения на комбинаторику с решениемв число Уравнения на комбинаторику с решениемвходит Уравнения на комбинаторику с решениемто разложение (1.8) примет вид:

Уравнения на комбинаторику с решением

Так что разложение Уравнения на комбинаторику с решениемна два сомножителя сводится к разделению каждого из чисел Уравнения на комбинаторику с решениемна две части, а это можно сделать Уравнения на комбинаторику с решениемспособами.

Ответ. Уравнения на комбинаторику с решением.

Пример №10

Сколькими способами легкоатлет, собираясь на тренировку, может выбрать себе пару спортивной обуви, имея 5 пар кроссовок и 2 нары кед?

Очевидно, что выбрать одну из имеющихся пар обуви, кроссовки или кеды, можно 5 + 2 = 7 способами.

Обобщая, приходим к комбинаторному правилу сложения:

  • если некоторый элемент Уравнения на комбинаторику с решениемможно выбрать Уравнения на комбинаторику с решениемспособами, а элемент Уравнения на комбинаторику с решением(независимо от выбора элемента Уравнения на комбинаторику с решением) — Уравнения на комбинаторику с решениемспособами, то выбрать Уравнения на комбинаторику с решениемилиУравнения на комбинаторику с решениемможно Уравнения на комбинаторику с решениемспособами.

Это правило справедливо также для трех и более элементов.

Пример №11

В меню школьной столовой предлагается на выбор 4 вида пирожков и 3 вида сока. Сколько разных вариантов выбора завтрака, состоящего из одного пирожка и одного стакана сока, имеется у учащегося этой школы? Уравнения на комбинаторику с решением

Пирожок можно выбрать 4 способами и к каждому пирожку выбрать сок 3 способами (рис. 76). Следовательно, учащийся имеет Уравнения на комбинаторику с решениемвариантов выбора завтрака.

Обобщая, приходим к комбинаторному правилу умножения:

  • если некоторый элемент Уравнения на комбинаторику с решениемможно выбрать Уравнения на комбинаторику с решением, способами и после каждого такого выбора (независимо от выбора элемента Уравнения на комбинаторику с решением) другой элемент Уравнения на комбинаторику с решениемможно выбрать Уравнения на комбинаторику с решениемспособами, то пару объектов Уравнения на комбинаторику с решениемиУравнения на комбинаторику с решениемможно выбрать Уравнения на комбинаторику с решениемспособами.

Это правило справедливо также для трех и более элементов.

Пример №12

Сколько трехзначных чисел можно составить из цифр 1, 2, 3, 4, если в числе: 1) цифры не повторяются; 2) цифры могут повторяться?

Уравнения на комбинаторику с решением

Решение:

1) Первую цифру можем выбрать 4 способами (рис.77). Так как после выбора первой цифры их останется три (ведь цифры в нашем случае повторяться не могут), то вторую цифру можем выбрать 3 способами.И наконец, третью цифру можем выбрать из оставшихся двух — то есть 2 способами. Следовательно, количество искомых трехзначных у чисел будет равно Уравнения на комбинаторику с решением.

2) Применим комбинаторное правило умножения. Так как цифры в числе могут повторяться, то каждую из цифр искомого числа можно выбрать 4 способами (рис. 78), и тогда таких чисел будет Уравнения на комбинаторику с решением.

Ответ. 1) 24 числа; 2) 64 числа.

Отметим, что решить подобные задачи без применения комбинаторного правила умножения можно только путем перебора всех возможных вариантов чисел, удовлетворяющих условию задачи. Но такой способ решения является слишком долгим и громоздким.

Пример №13

Сколько четных пятизначных чисел можно составить из цифр 5, 6, 7, 8, 9, если цифры в числе не повторяются?

Решение:

Четное пятизначное число можно получить, если последней его цифрой будет 6 или 8. Чисел, у которых последней является цифра 6, будет Уравнения на комбинаторику с решением(рис. 79),

Уравнения на комбинаторику с решением

а тех, у которых последней является цифра 8, — также 24. По комбинаторному правилу сложения всего четных чисел будет Уравнения на комбинаторику с решением.

Пример №14

Азбука племени АБАБ содержит всего две буквы — «а» и «б». Сколько слов в языке этого племени состоит: 1) из двух букв; 2) из трех букв?

Решение:

1) аа, ба, аб, бб (всего четыре слова); 2) ааа, ааб, аба, абб, ббб, бба, баб, баа (всего восемь слов).

Заметим, что найденное количество слов соответствует комбинаторному правилу умножения. Так как на каждое место есть два «претендента» — «а» и «б», то слов, состоящих из двух букв, будет Уравнения на комбинаторику с решением, а из трех букв — Уравнения на комбинаторику с решением.

Пример №15

В футбольной команде из 11 игроков надо выбрать капитана и его заместителя. Сколькими способами это можно сделать?

Решение:

Капитаном можно выбрать любого из 11 игроков, а его заместителем — любого из 10 оставшихся игроков. Таким образом (по правилу умножения), имеем Уравнения на комбинаторику с решениемразных способов.

Пример №16

В Стране Чудес 10 городов и каждые два из них соединяет авиалиния. Сколько авиалиний в этой стране?

Решение. Так как каждая авиалиния соединяет два города, то одним из них может быть любой из 10 городов, а другим — любой из 9 оставшихся. Следовательно, количество авиалиний равно Уравнения на комбинаторику с решением. Но при этом каждую из авиалиний мы учли дважды. Поэтому всего их будет Уравнения на комбинаторику с решением.

Комбинаторные задачи неразрывно связаны с задачами теории вероятностей, еще одного раздела математики.

В ХIII-ХII в. до н. э. встречаются упоминания о вопросах, близких к комбинаторным. Некоторые комбинаторные задачи решали и в Древней Греции. В частности, Аристоксен из Тарента (IV в. до н. э.), ученик Аристотеля, перечислил различные комбинации длинных и коротких слогов в стихотворных размерах. А Папп Александрийский в IV в. н. э. рассматривал число пар и троек, которые можно получить из трех элементов, допуская их повторения. Некоторые элементы комбинаторики были известны и в Индии во II в. до н. э. Индийцы умели вычислять числа, известные нам как коэффициенты формулы бинома Ньютона. Позднее, в VIII в. н. э., арабы нашли и саму эту формулу, и ее коэффициенты, которые сейчас вычисляют с помощью комбинаторных формул или «треугольника Паскаля».

Свой нынешний вид упомянутые комбинаторные формулы приобрели благодаря средневековому ученому Леви бен Гершону (XIV в.) и французскому математику П. Эригону (XVII в.).

В III в. н. э. сирийский философ Порфирий для классификации понятий составил специальную схему, получившую название «древо Порфирия». Сейчас подобные деревья используются для решения определенных задач комбинаторики в разнообразных областях знаний. Некоторые ранее неизвестные комбинаторные задачи рассмотрел Леонардо Пизанский (Фибоначчи) в своей знаменитой «Книге абака» (1202 г.), в частности, о нахождении наименьшего набора различных гирь, позволяющего взвесить груз с любой целочисленной массой, не превышающей заданного числа. Со времен греческих математиков были известны две последовательности, каждый член которых получали по определенному правилу из предыдущих, — арифметическая и геометрическая прогрессии. А Фибоначчи впервые в одной из задач выразил член последовательности через два предыдущих, используя формулу, которую назвали рекуррентной. В дальнейшем метод рекуррентных формул стал одним из мощнейших для решения комбинаторных задач.

Как ни странно, развитию комбинаторики в значительной степени способствовали азартные игры, которые были очень популярны в XVI в. В частности, вопросами определения разнообразных комбинаций в игре в кости в то время занимались такие известные итальянские математики, как Д. Кардано, H. Тарталья и др. А наиболее полно изучил этот вопрос в XVII в. Галилео Галилей.

Современные комбинаторные задачи высокого уровня сложности связаны с объектами в других отраслях математики: определителями, конечными геометриями, группами, математической логикой и т. п.

Правила суммы и произведения

Вспомните, что в математике любые совокупности называют множествами. Объекты, входящие в множества, называют его элементами. Множества обозначают большими латинскими буквами, а их элементы записывают в фигурных скобках. Считают, что все элементы множества различны.

Например, Уравнения на комбинаторику с решением

Множества бывают конечными и бесконечными. Если множество не содержит ни одного элемента, его называют пустым и обозначают символом Уравнения на комбинаторику с решением

Два множества называют равными, если они состоят из одних и тех же элементов.

Если Уравнения на комбинаторику с решением— часть множества Уравнения на комбинаторику с решениемто его называют подмножеством множества Уравнения на комбинаторику с решениеми записывают Уравнения на комбинаторику с решениемНаглядно это изображают с помощью диаграммы Эйлера (рис. 135, а). В частности, для числовых множеств правильные такие соотношения:

Уравнения на комбинаторику с решением

Случается, что множества Уравнения на комбинаторику с решениемимеют общие элементы. Если множество Уравнения на комбинаторику с решениемсодержит все общие элементы множеств Уравнения на комбинаторику с решениеми только их, то множество Уравнения на комбинаторику с решениемназывают пересечением множеств Уравнения на комбинаторику с решениемЗаписывают это так: Уравнения на комбинаторику с решениемДиаграммой Эйлера пересечение изображают, как показано на рисунке 135, б. Множество, содержащее каждый элемент каждого из множеств Уравнения на комбинаторику с решениеми только эти

Уравнения на комбинаторику с решением

элементы, называется объединением множеств Уравнения на комбинаторику с решениемЕсли Уравнения на комбинаторику с решением— объединение множеств Уравнения на комбинаторику с решениемто пишут Уравнения на комбинаторику с решением(рис. 135, в).

Разницей множеств Уравнения на комбинаторику с решениемназывают множество, состоящее из всех элементов множества Уравнения на комбинаторику с решениемне принадлежащих множеству Уравнения на комбинаторику с решениемЕго обозначают Уравнения на комбинаторику с решениемНапример, если Уравнения на комбинаторику с решениемУравнения на комбинаторику с решением

Говоря «множество», «подмножество», порядок их элементов не учитывают. Говорят, что они не упорядочены. Рассматривают и упорядоченные множества. Так называют множества с фиксированным порядком элементов. Их обозначают не фигурными, а круглыми скобками. Например, из элементов множества Уравнения на комбинаторику с решениемможно образовать 6 трёхэлементных упорядоченных множеств: Уравнения на комбинаторику с решением

Как множества, все они равны, как упорядоченные множества — разные.

Существуют задачи, в которых надо определить, сколько различных подмножеств или упорядоченных подмножеств можно образовать из элементов данного множества. Их называют комбинаторными задачами, а раздел математики, в котором рассматривается решение комбинаторных задач, называют комбинаторикой.

Комбинаторика — раздел математики, посвящённый решению задач выбора и расположения элементов некоторого конечного множества в соответствии с заданными правилами.

Рассмотрим два основных правила, с помощью которых решается много комбинаторных задач.

Пример №17

В городе Уравнения на комбинаторику с решениеместь два университета — политехнический и экономический. Абитуриенту нравятся три факультета в политехническом университете и два — в экономическом. Сколько возможностей имеет студент для поступления в университет?

Решение:

Обозначим буквой Уравнения на комбинаторику с решениеммножество факультетов, которые выбрал абитуриент в политехническом университете, а буквой Уравнения на комбинаторику с решением— в экономическом: Уравнения на комбинаторику с решениемПоскольку эти множества не имеют общих элементов, то в делом абитуриент имеет Уравнения на комбинаторику с решениемвозможностей для поступления в университет.

Описанную ситуацию можно обобщить в виде утверждения, которое называется правилом суммы.

Если элемент некоторого множества Уравнения на комбинаторику с решениемможно выбрать Уравнения на комбинаторику с решениемспособами, а элемент множества Уравнения на комбинаторику с решениемспособами, то элемент из множества Уравнения на комбинаторику с решениемили из множества Уравнения на комбинаторику с решениемможно выбрать Уравнения на комбинаторику с решениемспособами.

Правило суммы распространяется и на большее количество множеств.

Пример №18

Планируя летний отдых, семья определилась с местами его проведения: в Одессе — 1, в Евпатории — 3, в Ялте — 2, в Феодосии — 2. Сколько возможностей выбора летнего отдыха имеет семья?

Решение:

Поскольку все базы отдыха разные, то для решения задачи достаточно найти сумму элементов всех множеств, о которых говорится: Уравнения на комбинаторику с решениемСледовательно, семья может выбирать отдых из 8 возможных.

Пример №19

От пункта Уравнения на комбинаторику с решениемдо пункта Уравнения на комбинаторику с решениемведут три тропинки, а от Уравнения на комбинаторику с решением— две. Сколько маршрутов можно проложить от пункта Уравнения на комбинаторику с решениемдо пункта Уравнения на комбинаторику с решением

Решение:

Чтобы пройти от пункта Уравнения на комбинаторику с решениемдо пункта Уравнения на комбинаторику с решениемнадо выбрать одну из трёх тропинок: 1, 2 или 3 (рис. 136). После этого следует выбрать одну из двух других троп: 4 или 5. Всего от пункта Уравнения на комбинаторику с решениемдо пункта Уравнения на комбинаторику с решениемведут 6 маршрутов, потому что Уравнения на комбинаторику с решениемВсе эти маршруты можно обозначить с помощью пар:Уравнения на комбинаторику с решением

Обобщим описанную ситуацию.

Если первый компонент пары можно выбрать Уравнения на комбинаторику с решениемспособами, а . второй — Уравнения на комбинаторику с решениемспособами, то такую пару можно выбрать Уравнения на комбинаторику с решениемспособами.

Это — правило произведения, его часто называют основным правилом комбинаторики. Обратите внимание: речь идёт об упорядоченных парах, составленных из различных компонентов.

Правило произведения распространяется и на упорядоченные тройки, четвёрки и любые другие упорядоченные конечные множества. В частности, если первый компонент упорядоченной тройки можно выбрать Уравнения на комбинаторику с решениемспособами, второй — Уравнения на комбинаторику с решениемспособами, третий — Уравнения на комбинаторику с решениемспособами, то такую упорядоченную тройку можно выбрать Уравнения на комбинаторику с решениемспособами. Например, если столовая на обед приготовила 2 первых блюда — борщ (б) и суп (с ), 3 вторых — котлеты (к), вареники (в), голубцы (г) и 2 десертных — пирожные (п) и мороженое (м), то всего из трёх блюд столовая может предложить 12 различных наборов, поскольку Уравнения на комбинаторику с решением

Уравнения на комбинаторику с решением

Уравнения на комбинаторику с решением

Описанной ситуации соответствует диаграмма, изображённая на рисунке 137. Такие диаграммы называют деревьями.

Пример №20

Сколько разных поездов можно составить из 6 вагонов, если каждый из вагонов можно поставить на любом месте?

Решение:

Первым можно поставить любой из б вагонов. Имеем 6 выборов. Второй вагон можно выбрать из оставшихся 5 вагонов. Поэтому, согласно правилу умножения, два первых вагона можно выбрать Уравнения на комбинаторику с решениемспособами. Третий вагон можно выбрать из 4 вагонов, которые остались. Поэтому три первых вагона можно выбрать Уравнения на комбинаторику с решениемспособами. Продолжая подобные рассуждения, приходим к ответу: всего можно составить Уравнения на комбинаторику с решениемразличных поездов.

Обратите внимание на решение последней задачи. Оно свелось к вычислению произведения всех натуральных чисел от 1 до 6. В комбинаторике подобные произведения вычисляют часто.

Произведение всех натуральных чисел от 1 до Уравнения на комбинаторику с решениемназывают Уравнения на комбинаторику с решениемфакториалом и обозначают Уравнения на комбинаторику с решением

Уравнения на комбинаторику с решением

Условились считать, что Уравнения на комбинаторику с решением

Языком теории множеств правила суммы и произведения можно сформулировать следующим образом.

Если пересечение множеств Уравнения на комбинаторику с решениемпустое, то количество элементов в их объединении Уравнения на комбинаторику с решениемравно сумме количества элементов множеств Уравнения на комбинаторику с решением

Уравнения на комбинаторику с решением

Если множества Уравнения на комбинаторику с решениемимеют общие элементы, то

Уравнения на комбинаторику с решением

Если множества Уравнения на комбинаторику с решениемконечны, то количество возможных пар Уравнения на комбинаторику с решениемравно произведению количества элементов множеств Уравнения на комбинаторику с решением

Уравнения на комбинаторику с решением

Пример №21

В розыгрыше на первенство города по баскетболу принимают участие команды из 12 школ. Сколькими способами могут быть распределены первое и второе места?

Решение:

Первое место может получить одна из 12 команд. После того, как определён обладатель первого места, второе место может получить одна из 11 команд. Следовательно, общее количество способов, которыми можно распределить первое и второе места, равно Уравнения на комбинаторику с решением

Пример №22

Сколько четырёхзначных чисел можно составить из цифр 0,1, 2, 3, 4, 5, если ни одна цифра не повторяется?

Решение:

Первой цифрой числа может быть одна из 5 цифр 1, 2, 3, 4, 5. Если первая цифра выбрана, то вторая может быть выбрана 5-ю способами, третья — 4-мя, четвёртая — 3-мя. Согласно правилу умножения общее число способов равно:

Уравнения на комбинаторику с решением

Пример №23

Упростите выражение Уравнения на комбинаторику с решением

Решение:

Уравнения на комбинаторику с решениемУравнения на комбинаторику с решением

Размещения и перестановки

Задача:

Сколькими способами собрание из 20 человек может избрать председателя и секретаря?

Решение:

Председателя можно выбрать 20-ю способами, секретаря — из остальных 19 человек — 19-ю способами. По правилу произведения председателя и секретаря собрания могут выбрать Уравнения на комбинаторику с решениемспособами.

Обобщим задачу. Сколько упорядоченных Уравнения на комбинаторику с решениемэлементных подмножеств можно составить из Уравнения на комбинаторику с решениемразличных элементов? На первое место можно поставить любой из данных Уравнения на комбинаторику с решениемэлементов. На второе место — любой из остальных Уравнения на комбинаторику с решениемэлементов и т. д. На последнее Уравнения на комбинаторику с решениемместо можно поставить любой из остальных Уравнения на комбинаторику с решениемэлементов. Из правила произведения следует, что из данных Уравнения на комбинаторику с решениемэлементов можно получить Уравнения на комбинаторику с решениемУравнения на комбинаторику с решением-элементных упорядоченных подмножеств.

Например, из 4 элементов Уравнения на комбинаторику с решениемупорядоченных двухэлементных подмножеств можно образовать всего Уравнения на комбинаторику с решениемУравнения на комбинаторику с решением

Упорядоченое Уравнения на комбинаторику с решением-элементное подмножество Уравнения на комбинаторику с решениемэлементного множества называют размещением из Уравнения на комбинаторику с решениемэлементов Уравнения на комбинаторику с решением Их число обозначают Уравнения на комбинаторику с решением

Из предыдущих рассуждений следует, что Уравнения на комбинаторику с решениеми что для любых натуральных Уравнения на комбинаторику с решением

Уравнения на комбинаторику с решением

В правой части этого равенства Уравнения на комбинаторику с решениеммножителей. Поэтому результат можно сформулировать в виде такого утверждения.

Число размещений из Уравнения на комбинаторику с решениемэлементов по Уравнения на комбинаторику с решениемравно произведению Уравнения на комбинаторику с решениемпоследовательных натуральных чисел, наибольшее из которых Уравнения на комбинаторику с решением

Примеры:

Уравнения на комбинаторику с решением

Пример №24

Сколькими способами можно составить дневное расписание из пяти разных уроков, если класс изучает 10 различных предметов?

Решение:

Речь идёт об упорядоченных 5-элементных подмножествах некоторого множества, состоящего из 10 элементов.

Это размещения. Уравнения на комбинаторику с решением

Ответ. 30 240 способами.

Число размещений из Уравнения на комбинаторику с решениемэлементов по Уравнения на комбинаторику с решениемможно вычислять и по другой формуле: Уравнения на комбинаторику с решением(проверьте самостоятельно).

Размещение Уравнения на комбинаторику с решениемэлементов по Уравнения на комбинаторику с решениемназывают перестановками из Уравнения на комбинаторику с решениемэлементов. Их число обозначают Уравнения на комбинаторику с решением

Например, из трёх элементов Уравнения на комбинаторику с решениемможно образовать 6 различных перестановок: Уравнения на комбинаторику с решениемСледовательно, Уравнения на комбинаторику с решением

Подставив в формулу числа размещений Уравнения на комбинаторику с решениемполучим, что Уравнения на комбинаторику с решением

Число перестановок из Уравнения на комбинаторику с решениемэлементов равно Уравнения на комбинаторику с решением!

Примеры:

Уравнения на комбинаторику с решением

Пример №25

Сколькими способами можно составить список из 10 фамилий?

Решение:

Уравнения на комбинаторику с решением

Ответ. 3 628 800 способами.

Некоторые комбинаторные задачи сводятся к решению уравнений, в которых переменная указывает на количество элементов в некотором множестве или подмножестве. Рассмотрим несколько таких уравнений.

Пример №26

Решите уравнение Уравнения на комбинаторику с решением

Решение:

Пользуясь формулой размещений, данное уравнение можно заменить таким:

Уравнения на комбинаторику с решением

По условию задачи Уравнения на комбинаторику с решением— натуральное число, поэтому Уравнения на комбинаторику с решением— посторонний корень. Следовательно, Уравнения на комбинаторику с решением

Пример №27

Решите уравнение Уравнения на комбинаторику с решением

Решение:

Запишем выражения Уравнения на комбинаторику с решениемчерез произведения.

Имеем: Уравнения на комбинаторику с решением

Поскольку по смыслу задачи Уравнения на комбинаторику с решениемПоэтому последнее уравнение можно сократить на произведение Уравнения на комбинаторику с решениемТогда Уравнения на комбинаторику с решением Уравнения на комбинаторику с решениемНо уравнение Уравнения на комбинаторику с решениемудовлетворяет только одно значение: Уравнения на комбинаторику с решением

Пример №28

Команда из трёх человек выступает в соревнованиях по художественной гимнастике, в которых принимают участие ещё 27 спортсменок. Сколькими способами могут распределиться места между членами команды, при условии, что на этих соревнованиях ни одно место не делится?

Решение:

Речь идёт об упорядоченных 3-элементных подмножествах множества, состоящего из 30 элементов. Это — размещения. Уравнения на комбинаторику с решением

Пример №29

Сколькими способами можно разместить на полке 5 дисков?

Решение:

Речь идёт об упорядоченных 5-элементных множествах. Искомое количество способов равно Уравнения на комбинаторику с решением

Ответ. 120 способами.

Пример №30

Изображённое на рисунке 140 кольцо раскрашено в 7 цветов. Сколько существует таких колец, раскрашенных теми же цветами только в других последовательностях?

Решение:

Зафиксируем одну какую-нибудь часть кольца, окрашенную одним цветом, б других частей можно раскрасить Уравнения на комбинаторику с решениемспособами.

Уравнения на комбинаторику с решением

Ответ. 720 колец.

Пример №31

Сколько можно составить различных неправильных дробей, числителями и знаменателями которых есть числа 3,5, 7,9,11,13?

Решение:

Способ 1. Дробей, у которых числитель не равен знаменателю, можно составить Уравнения на комбинаторику с решениемто есть Уравнения на комбинаторику с решениемИз этих дробей только половина — неправильных, то есть — 15.

Неправильными являются также дроби, у которых числитель равен знаменателю. Таких дробей в нашем случае 6. Итак, всего можно составить Уравнения на комбинаторику с решением(дробь).

Способ 2. Если знаменатель неправильной дроби 3, то его числителями могут быть все 6 данных чисел. Если знаменатель 5, то числителями неправильной дроби могут быть 5 чисел (5, 7, 9, 11, 13) и т.д. Наконец, если знаменатель — число 13, то существует только 1 неправильная дробь, со знаменателем 13. Всего таких неправильных дробей существует Уравнения на комбинаторику с решением

Уравнения на комбинаторику с решением

Комбинации и бином ньютона

Пусть дано множество из трёх элементов: Уравнения на комбинаторику с решениемЕго двухэлементных подмножеств (не упорядоченных) существует всего три: Уравнения на комбинаторику с решениемГоворят, что существует 3 комбинации из трёх элементов по два. Пишут: Уравнения на комбинаторику с решением

Комбинацией из Уравнения на комбинаторику с решением элементов по Уравнения на комбинаторику с решением называют любое Уравнения на комбинаторику с решениемэлементное подмножество Уравнения на комбинаторику с решениемэлементного множества.

Число комбинаций из Уравнения на комбинаторику с решениемэлементов по Уравнения на комбинаторику с решениемобозначают Уравнения на комбинаторику с решениемВ отличие от размещений, комбинации — подмножества неупорядоченные.

Сравните: Уравнения на комбинаторику с решениемПри тех же значениях Уравнения на комбинаторику с решениемзначение Уравнения на комбинаторику с решениемменьше Уравнения на комбинаторику с решениемМожно также указать, во сколько раз меньше. Каждую Уравнения на комбинаторику с решениемэлементную комбинацию можно упорядочить Уравнения на комбинаторику с решениемспособами. В результате из одной комбинации получают Уравнения на комбинаторику с решениемразмещений (упорядоченных подмножеств) из тех же элементов. Итак,

число Уравнения на комбинаторику с решениемэлементных комбинаций в Уравнения на комбинаторику с решениемраз меньше числа размещений из тех же Уравнения на комбинаторику с решениемэлементов.

То есть, Уравнения на комбинаторику с решениемотсюда

Уравнения на комбинаторику с решением

Пример №32

Вычислите: Уравнения на комбинаторику с решением

Решение:

Уравнения на комбинаторику с решением

Обратите внимание! Уравнения на комбинаторику с решениемПолагают также, что Уравнения на комбинаторику с решениемдля любого Уравнения на комбинаторику с решением

Пример №33

Сколькими способами из 25 учеников можно выбрать на конференцию двух делегатов?

Решение:

Здесь Уравнения на комбинаторику с решениемпорядок учеников не имеет значения.

Уравнения на комбинаторику с решением

Ответ. 300-ми способами.

Докажем, что для натуральных значений Уравнения на комбинаторику с решениемправильно тождество Уравнения на комбинаторику с решением

Доказательство. Пусть дано Уравнения на комбинаторику с решениемразличных элементов: Уравнения на комбинаторику с решениемВсего из них можно образовать Уравнения на комбинаторику с решениемразличных Уравнения на комбинаторику с решениемэлементных комбинаций. Это количество комбинаций вычислим другим способом. Из данных Уравнения на комбинаторику с решениемэлементов, кроме последнего Уравнения на комбинаторику с решениемможно образовать Уравнения на комбинаторику с решениемкомбинаций. Остальные Уравнения на комбинаторику с решениемэлементные комбинации из всех данных элементов можно образовать, если к каждой комбинации из первых Уравнения на комбинаторику с решениемэлементов по Уравнения на комбинаторику с решениемдописать элемент Уравнения на комбинаторику с решениемТаких комбинаций Уравнения на комбинаторику с решением

Следовательно, Уравнения на комбинаторику с решениемА это и требовалось доказать.

Такое комбинаторное тождество можно доказать также, воспользовавшись формулой числа комбинаций.

С комбинациями тесно связана формула бинома Ньютона. Вспомните формулу квадрата двучлена: Уравнения на комбинаторику с решением

Умножив Уравнения на комбинаторику с решениемполучим формулы:

Уравнения на комбинаторику с решением

Эти три формулы можно записать и так:

Уравнения на комбинаторику с решением

Оказывается, для каждого натурального значения Уравнения на комбинаторику с решениемправильна и общая формула:

Уравнения на комбинаторику с решением

Это тождество называют формулой бинома Ньютона. а её правую часть разложением бинома Ньютона. Бином — латинское название двучлена. Пользуясь этой формулой, возведём, например, двучлен Уравнения на комбинаторику с решениемв пятую степень. Поскольку Уравнения на комбинаторику с решением

Уравнения на комбинаторику с решением

Доказать формулу бинома Ньютона можно методом математической индукции.

Доказательство. Предположим, что формула Уравнения на комбинаторику с решениемверна для некоторого натурального показателя степени Уравнения на комбинаторику с решениемПокажем, что тогда она верна и для следующего за ним значения Уравнения на комбинаторику с решением

Уравнения на комбинаторику с решением

Выражения в скобках преобразованы согласно формулы

Уравнения на комбинаторику с решением

Следовательно, если формула бинома Ньютона верна для Уравнения на комбинаторику с решениемто она правильна и для Уравнения на комбинаторику с решениемДля Уравнения на комбинаторику с решениемона правильна, так как Уравнения на комбинаторику с решениемПоэтому на основе аксиомы математической индукции можно утверждать, что формула верна для любого натурального показателя Уравнения на комбинаторику с решением

Вычислять коэффициенты разложения бинома Ньютона можно не по формуле числа комбинаций, а пользуясь числовым треугольником Паскаля — своеобразным способом вычисления коэффициентов разложения бинома Ньютона Уравнения на комбинаторику с решением

Уравнения на комбинаторику с решением

Треугольник Паскаля можно продолжать как угодно далеко. Это следует из тождества Уравнения на комбинаторику с решениемЕго крайние числа — единицы, а каждое другое равно сумме двух ближайших к нему чисел сверху.

Например, прибавляя числа шестой строки (для Уравнения на комбинаторику с решениемполучим числа следующей строки (для Уравнения на комбинаторику с решениемСледовательно, Уравнения на комбинаторику с решениемОбщий член разложения бинома Уравнения на комбинаторику с решениемможно определить по формуле Уравнения на комбинаторику с решением

  • первый член — Уравнения на комбинаторику с решением
  • второй член — Уравнения на комбинаторику с решением
  • третий член — Уравнения на комбинаторику с решением

Пример №34

В турнире по шашкам приняли участие 5 девушек и 7 юношей. Каждый участник сыграл один раз с каждым другим. Сколько партий было: а) между девушками; б) между юношами; в) между юношами и девушками?

Решение:

а) Речь идёт о 2-элементных подмножествах (неупорядоченных) множества, состоящего из 5 элементов. Это — комбинации. Уравнения на комбинаторику с решением

б) Аналогично Уравнения на комбинаторику с решением

в) Воспользуемся правилом умножения. Поскольку каждой из 5 девушек предстоит сыграть с каждым из 7 юношей, возможных случаев Уравнения на комбинаторику с решением

Пример №35

Для дежурства в столовой приглашают 3-х учеников из 7 класса и 2-х учеников из 10 класса. Сколькими способами это можно сделать, если в 7 классе учится 24 ученика, а в 10 классе — 18.

Решение:

Речь идёт о неупорядоченных подмножествах двух разных множеств. Это — комбинации.
Уравнения на комбинаторику с решением
По правилу произведения имеем Уравнения на комбинаторику с решениемспособов выбрать учащихся для дежурства.

Пример №36

Сколько разных делителей имеет число 1001?

Решение:

Разложим заданное число на простые множители: Уравнения на комбинаторику с решениемЕсли число Уравнения на комбинаторику с решением— делитель числа 1001, то оно должно быть одним из чисел 7, 11,13 (три случая) или любым их произведением. Различных произведений может быть Уравнения на комбинаторику с решениемДелителем данного числа есть ещё единица. Следовательно, число 1001 имеет Уравнения на комбинаторику с решениемделителей.

Пример №37

Докажите, что выпуклый Уравнения на комбинаторику с решениемугольник имеет Уравнения на комбинаторику с решениемдиагоналей.

Решение:

Отрезков, концами которых являются Уравнения на комбинаторику с решениемвершин данного Уравнения на комбинаторику с решением-угольника, существует Уравнения на комбинаторику с решениемСреди них есть и Уравнения на комбинаторику с решениемсторон данного Уравнения на комбинаторику с решением-угольника. Поэтому диагоналей он имеет Уравнения на комбинаторику с решениемУравнения на комбинаторику с решением

Пример №38

Уравнения на комбинаторику с решением

Решение:

Уравнения на комбинаторику с решением

Уравнения на комбинаторику с решением

Все члены разложения бинома Ньютона Уравнения на комбинаторику с решениемтакие же, как и члены разложения бинома Уравнения на комбинаторику с решениемтолько их члены с чётными номерами отрицательные.

Пример №39

Найдите номер члена разложения Уравнения на комбинаторику с решениемкоторый не содержит Уравнения на комбинаторику с решением

Решение:

Воспользуемся формулой общего члена разложения бинома. Имеем:

Уравнения на комбинаторику с решением

По условию задачи Уравнения на комбинаторику с решениемто есть Уравнения на комбинаторику с решениемОтсюда Уравнения на комбинаторику с решениемСледовательно, не содержит Уравнения на комбинаторику с решениемшестой член разложения бинома.

Видео:Комбинаторика: перестановка, размещение и сочетание | Математика | TutorOnlineСкачать

Комбинаторика: перестановка, размещение и сочетание | Математика | TutorOnline

Элементы комбинаторики

Решение многих задач теории вероятностей требует знания элементов комбинаторики, основными понятиями которой являются перестановки, размещения и сочетания.

Определение: Перестановки — это комбинации из одних и тех же элементов, отличающиеся только порядком элементов.

Пример:

Даны три числа 1, 2, 3. Определить количество комбинаций из этих элементов, отличающиеся только порядком элементов.

Решение:

Комбинации из данных элементов, отличающиеся только порядком элементов: 123; 132; 213; 231; 321; 312. Всего таких комбинаций Уравнения на комбинаторику с решениемЕсли дано n элементов, то число перестановок Уравнения на комбинаторику с решениемO2. Размещения — это комбинации, составленные из n различных элементов по m элементов, которые отличаются либо составом элементов, либо их расположением.

Пример:

Даны три числа 1, 2, 3. Определить количество размещений из этих элементов по два, отличающиеся составом или порядком элементов.

Решение:

Комбинации из данных элементов по два, отличающиеся составом или порядком элементов: 12; 21; 23; 32; 13; 31. Всего таких комбинаций 6. Если дано n элементов, то число размещений по m элементов, которые отличаются либо составом элементов, либо их расположением: Уравнения на комбинаторику с решением

Определение: Сочетания — это комбинации, составленные из n различных элементов по m элементов, которые отличаются друг от друга хотя бы одним элементом.

Пример:

Даны три числа 1, 2, 3. Определить количество размещений из этих элементов по два, отличающиеся хотя бы одним элементом.

Решение:

Комбинации из данных элементов по два, отличающиеся хотя бы одним элементом: 12; 23; 13. Всего таких комбинаций 3. Если дано n элементов, то число сочетаний по m элементов, которые отличаются хотя бы одним элементом:Уравнения на комбинаторику с решением

Пример:

Пусть в урне находится n прономерованных шаров. Определить количество способов, которыми можно извлечь из урны эти шары один за другим.

Решение:

Число способов равно числу различных комбинаций из п элементов, отличающихся только порядком элементов, т.е. числу перестановок: Уравнения на комбинаторику с решением

Пример:

Из колоды, содержащей 36 карт, наугад вынимают 3 карты. Найти вероятность того, что среди выбранных карт окажется один туз.

Решение:

Событие А состоит в том, что среди выбранных карт окажется один туз. Это сложное событие состоит из двух событий: выбирается один туз из четырех, а две другие карты выбираются из оставшихся 32 карт. Следовательно, число случаев, благоприятствующих появлению события A, равно Уравнения на комбинаторику с решениемВсего возможных равновероятных исходов, образующих полную группу определяется числом сочетаний из 36 карт по 3 карты, т.е. Уравнения на комбинаторику с решениемТаким образом, вероятность события А равна Уравнения на комбинаторику с решением

Арифметика случайных событий

Будем считать, что все события, которые могут произойти в рамках данного эксперимента, располагаются внутри квадрата G, тогда невозможные события располагаются вне квадрата G (Рис. 2): Уравнения на комбинаторику с решением

Рис. 2. Квадрат возможных событий.

Таким образом, достоверное событие определяется внутренней частью квадрата, а невозможное — областью вне квадрата.

Определение: Суммой двух случайных событий А и В называется третье случайное событие С, которое состоит в том, что произойдет (или не произойдет) или событие А, или событие В : С = А + В (Рис. 3).

Определение: Суммой n случайных событий Уравнения на комбинаторику с решениемназывается случайное событие С, которое реализуется в данном опыте, если произойдет (или не произойдет) или одно событий Уравнения на комбинаторику с решением, или любая их совокупность: Уравнения на комбинаторику с решением

Уравнения на комбинаторику с решением

Рис. 3. Сумма случайных событий

Замечание: Если в словесном описании сложного события присутствует разделительный союз “или” между элементарными событиями, то речь идет о сумме этих элементарных событий.

Замечание: Суммой события А и ему противоположного события Уравнения на комбинаторику с решениемявляется достоверное событие Уравнения на комбинаторику с решениемт.е. Уравнения на комбинаторику с решениемСледовательно, противоположное событие можно записать в виде Уравнения на комбинаторику с решением

Определение: Произведением двух случайных событий А и В называется третье случайное событие С, которое состоит в том, что произойдет (или не произойдет) и событие А, и событие В : Уравнения на комбинаторику с решением(Рис. 4). Уравнения на комбинаторику с решением

Рис. 4. Произведение случайных событий.

Определение: Произведением n случайных событий Уравнения на комбинаторику с решениемназывается случайное событие С, которое реализуется в данном опыте, если произойдет (или не произойдет) совместная реализация событий Уравнения на комбинаторику с решением

Замечание: Если в словесном описании сложного события присутствует соединительный союз “и” между элементарными событиями, то речь идет о произведении этих элементарных событий.

Пример №40

Пусть имеются передатчик и приемник. Приемник удален от передатчика недостаточно большое расстояние, при котором он может при определенных условиях не принять один из сигналов, переданных передатчиком. Пусть передатчик послал три сигнала. Определить следующие сложные события:

  • а) приемник принят только второй сигнал (событие А );
  • б) приемник принял только один сигнал (событие В);
  • в) приемник принял не менее двух сигналов (2 или 3 сигнала — событие С);
  • г) приемник не принял ни одного сигнала (событие D);
  • д) приемник принял хотя бы один сигнал (событие E).

Решение:

Обозначим через Уравнения на комбинаторику с решениемэлементарное событие, состоящее в том, что приемник принял сигнал i.

Сложное событие А состоит в том, что приемник не принял первый сигнал и принял второй сигнал, и не принял третий сигнал. Так как между элементарными событиями стоит соединительный союз “и”, то речь идет о их произведении, т.е. Уравнения на комбинаторику с решением

Сложное событие В состоит в том, что приемник принял или первый сигнал, или принял второй сигнал, или принял третий сигнал. Так как между элементарными событиями стоит разделительный союз “или”, то речь идет о сумме сложных событии, т.е. Уравнения на комбинаторику с решением

Рассуждая аналогично, получим выражения для остальных событий: Уравнения на комбинаторику с решениемСложное событие Е содержит в своем словесном описании слова “хотя бы один”, следовательно, оно противоположно событию, содержащему в своем словесном описании слова “ни один”, т.е. событию D: Уравнения на комбинаторику с решением

Теорема сложения вероятностей несовместных событий

Теорема: Если случайные события А и В несовместны, то вероятность их суммы равна сумме вероятностей этих событий, т.е. Р(А + В) = Р(А) + Р(В)

Доказательство: Пусть в данном опыте имеется n равновозможных, элементарных, несовместных событий и пусть в m случаях наступает событие А, а в l случаях-событие В. Тогда появлению события А + В благоприятствует m+l исходов. Поэтому Уравнения на комбинаторику с решением

Следствие: Если имеется N событий, то Уравнения на комбинаторику с решением

Следствие: Если события Уравнения на комбинаторику с решением(Уравнения на комбинаторику с решением) образуют полную группу, то Уравнения на комбинаторику с решением

Доказательство: Так как события Уравнения на комбинаторику с решениемобразуют полную группу равно возможных, элементарных, несовместных событий, то их сумма есть достоверное событие Уравнения на комбинаторику с решениема вероятность достоверного события равна 1.

Следствие: Вероятность суммы противоположных событий равна 1.

Доказательство: В силу того, что события А и ему противоположное событие Уравнения на комбинаторику с решениемобразуют полную группу несовместных событий, то по следствию вероятность их суммы равна 1.

Замечание: Если сложное событие состоит из суммы элементарных событий, то перед применением теоремы надо определить совместны или несовместны элементарные события.

Пример:

Пусть в урне находится 5 белых шаров, 3 — красных и 4 — зеленых. Из урны наудачу вынули шар. Какова вероятность того, что данный шар цветной?

Решение:

Событие, состоящее в том, что из урны извлечен красный шар, обозначим через А. Событие, состоящее в том, что из урны извлечен зеленый шар, обозначим через В. Тогда извлечение цветного шара есть событие С. Так как события А и В несовместны, т.е. событие С состоит в том, что из урны извлечен или событие А , или событие В, то С = А + В. Используя теорему о сложении вероятностей несовместных событий, получим:

Уравнения на комбинаторику с решением

Зависимые и независимые события. Условная и безусловная вероятности

Определение: Случайные события А и В называются независимыми, если появление одного из них не влияет на вероятность появления другого события, в противном случае события называются зависимыми.

Замечание: В этом определении речь идет не о причинно-следственной связи между событиями, а о вероятностной (появление одного из них не влияет на вероятность появления другого события), которая является более общей зависимостью между событиями.

Пример №41

В хранилище находится 10 исправных и 5 неисправных приборов, причем неизвестно, какие из них исправные, а какие — нет. Обозначим событием А — из хранилища взят исправный прибор, а В — взят неисправный прибор. Пусть вначале взят неисправный прибор. Определить вероятности указанных событий с возвращением неисправного прибора на склад и без возвращения неисправного прибора в хранилище.

Решение:

Если неисправный прибор возвращается в хранилище, то события А и В независимы и их вероятности равны Уравнения на комбинаторику с решениемВо втором случае, когда неисправный прибор не возвращается на склад, общее количество приборов в хранилище изменилось и стало равным 14, причем неисправных приборов будет храниться 4. Следовательно, произошедшее событие В изменило вероятности события А и В: Уравнения на комбинаторику с решениемт.е. при такой организации эксперимента события А и В являются зависимыми.

Определение: Вероятность случайного события называется безусловной, если при ее вычислении на комплекс условий, в которых рассматривается это случайное событие, не накладывается никаких дополнительных ограничений. Безусловная вероятность обозначается Уравнения на комбинаторику с решением

Определение: Вероятность случайного события называется условной, если она вычисляется при условии, что произошло другое случайное событие. Условная вероятность обозначается Уравнения на комбинаторику с решением

Теорема умножения вероятностей

Т.2. Вероятность совместного появления двух случайных событий А и В равна произведению вероятности одного из них на условную вероятность другого события, вычисленную при условии, что первое событие имело место: Уравнения на комбинаторику с решением

Доказательство: Пусть событие А состоит в том, что брошенная точка наугад в квадрат G попадает в область А, которая имеет площадь Уравнения на комбинаторику с решениемСобытие В состоит в том, что брошенная наугад в квадрат G точка попадает в область В с площадью Уравнения на комбинаторику с решениемПусть весь квадрат имеет площадь S, а область совместного наступления событий Уравнения на комбинаторику с решениемимеет площадь Уравнения на комбинаторику с решением(Рис. 5). Тогда вероятность события А равна Уравнения на комбинаторику с решениема события В — Уравнения на комбинаторику с решением

Уравнения на комбинаторику с решением

Рис. 5. Совместное наступление зависимых и независимых случайных событий.

Вероятность совместного наступления событий Уравнения на комбинаторику с решением.Условные вероятности того, что произойдут указанные события, определяются по формулам: Уравнения на комбинаторику с решениемТаким образом, можно записать, что вероятность совместного наступления событий Уравнения на комбинаторику с решениемравна:

Уравнения на комбинаторику с решением

Замечание: Если события А и В независимы, то Уравнения на комбинаторику с решениемт.е. безусловная и условная вероятности равны между собой.

В связи с вышеприведенным замечанием теорема об умножении вероятностей независимых случайных событий имеет вид:

ТЗ. Вероятность совместного наступления независимых событий равна произведению вероятностей этих событий: Уравнения на комбинаторику с решением

Замечание: Независимость случайных событий всегда взаимная. Если Уравнения на комбинаторику с решениемто по теореме Уравнения на комбинаторику с решениемоткуда следует, чтоУравнения на комбинаторику с решением

Следствие: Методом математической индукции теоремы легко обобщается на произведение N зависимых событий:

Уравнения на комбинаторику с решениема теорема — для независимых событий: Уравнения на комбинаторику с решением

Замечание: Если сложное событие представляется в виде произведения элементарных событий, то при вычислении вероятности такого события надо определить, зависимы или независимы эти элементарные события.

Видео:9 класс. Алгебра. Решение уравнений. Элементы комбинаторики.Скачать

9 класс. Алгебра. Решение уравнений. Элементы комбинаторики.

Что такое комбинаторика

Понятие множества и его элементов:

  • Элемент а принадлежит множеству АУравнения на комбинаторику с решениемУравнения на комбинаторику с решением
  • Элемент Уравнения на комбинаторику с решениемпринадлежит множеству Уравнения на комбинаторику с решениемУравнения на комбинаторику с решением
  • В множестве нет элементовУравнения на комбинаторику с решениемУравнения на комбинаторику с решением

Множество можно представить как совокупность некоторых объектов, объединенных по определенному признаку. В математике множество — одно из основных неопределяемых понятий. Каждый объект, принадлежащий множеству А, называется элементом этого множества. Множество, не содержащее ни одного элемента, называется пустым множеством и обозначается Уравнения на комбинаторику с решением.

ПодмножествоУравнения на комбинаторику с решением

Уравнения на комбинаторику с решением

Если каждый элемент множества А является элементом множества В, то говорят, что множество А является подмножеством множества В,

и записывают так: Уравнения на комбинаторику с решениемИспользуется также запись Уравнения на комбинаторику с решениемесли множество А или является подмножеством множества В, или равно множеству В.

Уравнения на комбинаторику с решением

Два множества называются равными, если каждый элемент первого множества является элементом второго множества и, наоборот, каждый элемент второго множества является элементом первого множества.

Пересечение множествУравнения на комбинаторику с решением

Уравнения на комбинаторику с решением

Пересечением множеств A и В называют их общую часть, то есть множество С всех элементов, принадлежащих как множеству А, так и множеству В

Объединение множеств Уравнения на комбинаторику с решением

Уравнения на комбинаторику с решением

Объединением множеств А и В называют множество С, состоящее из всех элементов, принадлежащих хотя бы одному из этих множеств (А или В)

Разность множеств Уравнения на комбинаторику с решением

Уравнения на комбинаторику с решением

Разностью множеств А и В называется множество С, которое состоит из всех элементов, принадлежащих множеству А и не принадлежащих множеству В

Уравнения на комбинаторику с решением

Если все рассматриваемые множества являются подмножествами некоторого универсального множества U, то разность U А называется дополнением множества А. Другими словами, дополнением множества А называется множество, состоящее из всех элементов, не принадлежащих множеству А (но принадлежащих универсальному множеству).

Объяснение и обоснование:

Понятие множества

Одним из основных понятий, которые используются в математике, является понятие множества. Для него не дается определения. Можно пояснить, что множеством называют произвольную совокупность объектов, а сами объекты — элементами данного множества. Так, можно говорить о множестве учеников в классе (элементы — ученики), множестве дней недели (элементы — дни недели), множестве натуральных делителей числа 6 (элементы — числа 1, 2, 3, 6) и т. д.

В курсах алгебры и алгебры и начал анализа чаще всего рассматривают множества, элементами которых являются числа, и поэтому их называют числовыми множествами.

Как правило, множества обозначают прописными буквами латинского алфавита. Например, если множество М состоит из чисел 1; 2; 3, то его обозначают так: М = . Тот факт, что число 2 входит в это множество (является элементом данного множества М) записывается с помощью специального значка Уравнения на комбинаторику с решениемследующим образом: Уравнения на комбинаторику с решением; а то, что число 5 не входит в это множество (не является элементом данного множества), записывается так:Уравнения на комбинаторику с решением

Можно рассматривать также множество, не содержащее ни одного элемента, — пустое множество.

Например: множество простых делителей числа 1 — пустое множество.

Для некоторых множеств существуют специальные обозначения. Так, пустое множество обозначается символомУравнения на комбинаторику с решением, множество всех натуральных чисел — буквой N, множество всех целых чисел — буквой Z, множество всех рациональных чисел — буквой Q, а множество всех действительных чисел — буквой R.

Множества бывают конечными и бесконечными в зависимости от того, какое количество элементов они содержат. Так, множества А = и М = — конечные потому, что содержат конечное число элементов, а множества N, Z, Q, R — бесконечные.

Множества задают или с помощью перечисления их элементов (это можно сделать только для конечных множеств), или с помощью описания, когда задается правило (характеристическое свойство), которое позволяет определить, принадлежит или нет данный объект рассматриваемому множеству. Например, А = (множество задано перечислением элементов), В — множество четных целых чисел (множество задано характеристическим свойством элементов множества). Последнее множество иногда записывают так: Уравнения на комбинаторику с решением— четное целое число> или так: Уравнения на комбинаторику с решением— здесь после вертикальной черточки записано характеристическое свойство.

В общем виде запись множества с помощью характеристического свойства можно обозначить так: Уравнения на комбинаторику с решением— характеристическое свойство. Например,Уравнения на комбинаторику с решением

Равенство множеств

Пусть А — множество цифр трехзначного числа 312, то есть А = , а В — множество натуральных чисел, меньших четырех, то есть В = . Поскольку эти множества состоят из одних и тех же элементов, то они считаются равными. Это записывают так: А = В.

Для бесконечных множеств таким способом (сравнивая все элементы) установить их равенство невозможно. Поэтому в общем случае равенство множеств определяется следующим образом.

Два множества называются равными, если каждый элемент первого множества является элементом второго множества и, наоборот, каждый элемент второго множества является элементом первого множества.

Из приведенного определения равенства множеств следует, что в множестве одинаковые элементы не различаются. Действительно, например, = , поскольку каждый элемент первого множества (1 или 2) является элементом второго множества и, наоборот, каждый элемент второго множества (1 или 2) является элементом первого. Поэтому, записывая множество, чаще всего каждый его элемент записывают только один раз.

Подмножество

Если каждый элемент множества А является элементом множества В, то говорят, что множество А является подмножеством множества В.

Это записывают следующим образом: Уравнения на комбинаторику с решением

Например, Уравнения на комбинаторику с решением(поскольку любое натуральное число — целое), Уравнения на комбинаторику с решением(поскольку любое целое число — рациональное), Уравнения на комбинаторику с решением(поскольку любое рациональное число — действительное).

Полагают, что всегдаУравнения на комбинаторику с решением, то есть пустое множество является подмножеством любого множества.

Иногда вместо записи Уравнения на комбинаторику с решениемиспользуется также запись Уравнения на комбинаторику с решением, если множество А является подмножеством множества В или равно множеству В. Например, можно записать, что Уравнения на комбинаторику с решением.

Сопоставим определение равенства множеств с определением подмножества. Если множества А и В равны, то: 1) каждый элемент множества А является элементом множества В, следовательно, А — подмножество ВУравнения на комбинаторику с решением; 2) каждый элемент множества В является элементом множества А, следовательно, В — подмножество Уравнения на комбинаторику с решениемТаким образом,

два множества равны, если каждое из них является подмножеством другого.

А = В означает то же, что Уравнения на комбинаторику с решением

Иногда соотношения между множествами удобно иллюстрировать с помощью кругов (которые часто называют кругами Эйлера-Венна). Например, рисунок 118 иллюстрирует определение подмножества, а рисунок 119-отношения между множествами Уравнения на комбинаторику с решением

Уравнения на комбинаторику с решением

Операции над множествами

Над множествами можно выполнять определенные действия: находить их пересечение, объединение, разность. Дадим определение этих операций и проиллюстрируем их с помощью кругов.

Пересечением множеств А и В называют их общую часть, то есть множество С всех элементов, принадлежащих как множеству А, так и множеству В.

Пересечение множеств обозначают знаком Уравнения на комбинаторику с решением(на рисунке 120 приведена иллюстрация и символическая запись определения пересечения множеств).

Например, если А = , В = , то Уравнения на комбинаторику с решением

Объединением множеств А и В называют множество С, состоящее из всех элементов, принадлежащих хотя бы одному из этих множеств (А или В).

Объединение множеств обозначают знаком U (на рисунке 121 приведена иллюстрация и символическая запись определения объединения множеств).

Например, для множеств А и В из предыдущего примера Уравнения на комбинаторику с решениемЕсли обозначить множество иррациональных чисел через М, то М U Q = R. Разностью множеств А и В называется множество С, состоящее из всех элементов, которые принадлежат множеству А и не принадлежат множеству В.

Разность множеств обозначают знаком . На рисунке 122 приведена иллюстрация и символическая запись определения разности множеств.

Например, если А = , В = , то АВ = , а В А = . Если В — подмножество А, то разность А В называют дополнением множества В до множества А (рис. 123).

Например, если обозначить множество иррациональных чисел через М, то R Q = М: множество М иррациональных чисел дополняет множество Q рациональных чисел до множества R всех действительных чисел.

Все множества, которые мы рассматриваем, являются подмножествами некоторого так называемого универсального множества U. Его обычно изображают в виде прямоугольника, а все остальные множества — в виде кругов внутри этого прямоугольника (рис. 124). Разность U А называется дополнением множества А. Уравнения на комбинаторику с решением

Уравнения на комбинаторику с решением

Дополнением множества А называется множество, состоящее из всехэлементов, не принадлежащих множеству А (но принадлежащих универсальному множеству U).

Дополнение множества А обозначается Уравнения на комбинаторику с решением(можно читать: «А с чертой»). Например, если U = R и А = [0; 1], то Уравнения на комбинаторику с решениемДля этого примера удобно использовать традиционную иллюстрацию множества действительных чисел на числовой прямой (рис. 125).

Видео:Уравнение, комбинаторика, сочетания, факториалы | Это how? #5Скачать

Уравнение, комбинаторика, сочетания, факториалы | Это how? #5

Комбинаторика и Бином Ньютона

Элементы комбинаторики:

Комбинаторика — раздел математики, в котором изучаются способы выбора и размещения элементов некоторого конечного множества на основании некоторых условий. Выбранные (или выбранные и размещенные) группы элементов называются Соединения с повторениямими.

Если все элементы полученного множества разные — получаем соединения без повторений, а если в полученном множестве элементы повторяются, то получаем соединения с повторениями*.

Перестановкой из п элементов называется любое упорядоченное множество из Уравнения на комбинаторику с решениемэлементов.

Иными словами, это такое множество, для которого указано, какой элемент находится на первом месте, какой — на втором. какой — на п-м.

*Формулы для нахождения количества соединений с повторениями являются обязательными только для классов физико-математического профиля. Формула числа перестановок Уравнения на комбинаторику с решением Уравнения на комбинаторику с решением(читается: «Эн факториал»)

Количество различных шестизначных чисел, которые можно составить из цифр 1, 2, 3, 4, 5, 6, не повторяя эти цифры в одном числе, равно Уравнения на комбинаторику с решением

Размещением из Уравнения на комбинаторику с решениемэлементов по Уравнения на комбинаторику с решениемназывается любое упорядоченное множество из Уравнения на комбинаторику с решениемэлементов, состоящее из элементов Уравнения на комбинаторику с решением-элементного множества Формула числа размещенийУравнения на комбинаторику с решением

Уравнения на комбинаторику с решением

Количество различных трехзначных чисел, которые можно составить из цифр 1,2,3, 4, 5, 6, если цифры не могут повторяться, равно Уравнения на комбинаторику с решением

Сочетанием без повторений из Уравнения на комбинаторику с решениемэлементов по Уравнения на комбинаторику с решениемназывается любое Уравнения на комбинаторику с решением-элементное подмножество Уравнения на комбинаторику с решением-элементного множества Формула числа сочетанийУравнения на комбинаторику с решением

Уравнения на комбинаторику с решением(по определению считают, что Уравнения на комбинаторику с решением)

Из класса, состоящего из 25 учащихся, можно выделить 5 учащихся для дежурства по школе Уравнения на комбинаторику с решениемспособами, то есть Уравнения на комбинаторику с решениемспособами. Некоторые свойства числа сочетаний без повторений Уравнения на комбинаторику с решением

Схема решения комбинаторных задач

Если элемент А можно выбрать Уравнения на комбинаторику с решениемспособами, а элемент В — Уравнения на комбинаторику с решениемспособами, то А или В можно выбрать Уравнения на комбинаторику с решениемспособами.

Если элемент А можно выбрать Уравнения на комбинаторику с решениемспособами, а после этого элемент В — Уравнения на комбинаторику с решениемспособами, то А и В можно выбрать Уравнения на комбинаторику с решениемспособами. Выбор формулы

Учитывается ли порядок следования элементов в соединении?

Все ли элементы входят в соединение?

без повторений с повторениями без повторений с повторениями без повторений с повторениямиУравнения на комбинаторику с решением

Объяснение и обоснование:

Понятие соединения

При решении многих практических задач приходится выбирать из определенной совокупности объектов элементы, имеющие те или иные свойства, размещать эти элементы в определенном порядке и т. д. Поскольку в этих задачах речь идет о тех или иных комбинациях объектов, то такие задачи называют комбинаторными. Раздел математики, в котором рассматриваются методы решения комбинаторных задач, называется комбинаторикой. В комбинаторике рассматривается выбор и размещение элементов некоторого конечного множества на основании определенных условий.

Выбранные (или выбранные и размещенные) группы элементов называют соединениями. Если все элементы полученного множества разные — получаем размещения без повторений, а если в полученном множестве элементы могут повторяться, то получаем размещения с повторениями. Рассматриваются соединения без повторений, а соединения с повторениями.

Решение многих комбинаторных задач базируется на двух основных правилах — правиле суммы и правиле произведения.

Правило суммы

Если на тарелке лежит 5 груш и 4 яблока, то выбрать один фрукт (то есть грушу или яблоко) можно 9 способами (5 + 4 = 9). В общем виде имеет место такое утверждение:

  • если элемент А можно выбрать Уравнения на комбинаторику с решениемспособами, а элемент В — Уравнения на комбинаторику с решениемспособами, то А или В можно выбрать Уравнения на комбинаторику с решениемспособами.

Правило произведения

Если в киоске продают ручки 5 видов и тетради 4 видов, то выбрать набор из ручки и тетради (то есть пару — ручка и тетрадь) можно 5 • 4 = 20 способами (поскольку с каждой из 5 ручек можно взять любую из 4 тетрадей). В общем виде имеет место такое утверждение:

  • если элемент А можно выбрать m способами, а после этого элемент В — Уравнения на комбинаторику с решениемспособами, то А и В можно выбрать m • п способами.

Это утверждение означает, что если для каждого из т элементов А можно взять в пару любой из Уравнения на комбинаторику с решениемэлементов В, то количество пар равно произведению Уравнения на комбинаторику с решением

Повторяя приведенные рассуждения несколько раз (или, иначе говоря, используя метод математической индукции), получаем, что правила суммы и произведения можно применять при выборе произвольного конечного количества элементов.

Следовательно, если приходится выбирать или первый элемент, или второй, или третий и т. д. элемент, количества способов выбора каждого еле-мента складывают, а когда приходится выбирать набор, в который входят и первый, и второй, и третий, и т. д. элементы, количества способов выбора каждого элемента перемножают.

Упорядоченные множества

При решении комбинаторных задач приходится рассматривать не только множества, в которых элементы можно записывать в любом порядке, но и так называемые упорядоченные множества. Для упорядоченных множеств существенным является порядок следования их элементов, то есть то, какой элемент записан на первом месте, какой на втором и т. д. В частности, если одни и те же элементы записать в разном порядке, то мы получим различные упорядоченные множества. Чтобы различить записи упорядоченного и неупорядоченного множеств, элементы упорядоченного множества часто записывают в круглых скобках, например Уравнения на комбинаторику с решением

Рассматривая упорядоченные множества, следует учитывать, что упорядоченность не является свойством самого неупорядоченного множества (из которого мы получили упорядоченное), поскольку одно и то же множество можно по-разному упорядочить. Например, множество из трех чисел можно упорядочить по возрастанию: (-5; 1; 3), по убыванию: (3; 1; — 5), по возрастанию абсолютной величины числа: (1; 3; -5) и т. д.

Будем понимать, что для того чтобы задать конечное упорядоченное множество из п элементов, достаточно указать, какой элемент находится на первом месте, какой на втором, . какой на п-м.

Размещения

Размещением из Уравнения на комбинаторику с решениемэлементов по Уравнения на комбинаторику с решениемназывается любое упорядоченное множество из Уравнения на комбинаторику с решениемэлементов, состоящее из элементов Уравнения на комбинаторику с решением-элементного множества.

Например, из множества, содержащего три цифры , можно составить следующие размещения из двух элементов без повторений: (1;5),(1;7),(5; 7), (5; 1), (7; 1), (7; 5).

Количество размещений из Уравнения на комбинаторику с решениемэлементов по Уравнения на комбинаторику с решениемобозначается Уравнения на комбинаторику с решением(читается: «А из Уравнения на комбинаторику с решениемпо Уравнения на комбинаторику с решением», А — первая буква французского слова arrangement, что означает «размещение, приведение в порядок»). Как видим,Уравнения на комбинаторику с решением

Выясним, сколько всего можно составить размещений из Уравнения на комбинаторику с решениемэлементов по Уравнения на комбинаторику с решениембез повторений. Составление размещения представим себе как последовательное заполнение Уравнения на комбинаторику с решениеммест, которые мы будем изображать в виде клеточек (рис. 126). На первое место мы можем выбрать один из п элементов заданного множества (то есть элемент для первой клеточки можно выбрать Уравнения на комбинаторику с решениемспособами).

Если элементы нельзя повторять, то на второе место можно выбрать только один элемент из оставшихся, то есть из Уравнения на комбинаторику с решением— 1 элементов. Теперь уже два элемента использованы и на третье место можно выбрать только один из Уравнения на комбинаторику с решением— 2 элементов и т. д. На Уравнения на комбинаторику с решением-e место можно выбрать только один из Уравнения на комбинаторику с решениемэлементов.

Поскольку требуется выбрать элементы и на первое место, и на второе, . и наУравнения на комбинаторику с решением-e, то используем правило произведения, получим следующую формулу числа размещений из Уравнения на комбинаторику с решениемэлементов по Уравнения на комбинаторику с решениемУравнения на комбинаторику с решением

Например, Уравнения на комбинаторику с решением(что совпадает с соответствующим значением, полученным выше). Аналогично можно обосновать формулу для нахождения числа размещений с повторениями.

При решении простейших комбинаторных задач важно правильно выбрать формулу, по которой будут проводиться вычисления. Для этого достаточно выяснить следующее: Уравнения на комбинаторику с решением

  • — Учитывается ли порядок следования элементов в соединении?
  • — Все ли заданные элементы входят в полученное соединение?

Если, например, порядок следования элементов учитывается и из Уравнения на комбинаторику с решениемзаданных элементов в соединении используется только Уравнения на комбинаторику с решениемэлементов, то по определению — это размещение из Уравнения на комбинаторику с решениемэлементов по Уравнения на комбинаторику с решением.

Заметим, что после определения вида соединения следует также выяснить, могут ли элементы в соединении повторяться, то есть выяснить, какую формулу необходимо использовать — для количества соединений без повторений или с повторениями.

Примеры решения задач:

Пример №42

На соревнования по легкой атлетике приехала команда из 12 спортсменок. Сколькими способами тренер может определить, кто из них побежит в эстафете 4 х 100 м на первом, втором, третьем и четвертом этапах?

Решение:

Уравнения на комбинаторику с решениемКоличество способов выбрать из 12 спортсменок четырех для участия в эстафете равно количеству размещений из 12 элементов по 4 (без повторений), то есть Уравнения на комбинаторику с решением

Для выбора формулы выясняем ответы на вопросы, приведенные выше. Поскольку для спортсменок важно, в каком порядке они будут бежать, то порядок при выборе элементов учитывается. В полученное соединение входят не все 12 заданных элементов. Следовательно, соответствующее соединение — размещение из 12 элементов по 4 (без повторений, поскольку каждая спортсменка может бежать только на одном этапе эстафеты).

Пример №43

Найдите количество трехзначных чисел, которые можно составить из цифр 1, 2, 3, 4, 5, 6, 7, если цифры в числе не повторяются.

Решение:

Уравнения на комбинаторику с решениемКоличество трехзначных чисел, которые можно составить из семи цифр 1, 2, 3, 4, 5, 6, 7, равно числу размещений из 7 элементов по 3, то есть

Уравнения на комбинаторику с решением

Для выбора формулы выясняем, что для чисел, которые мы будем составлять, порядок следования цифр учитывается и не все элементы выбираются (только 3 из заданных семи). Следовательно, соответствующее соединение — размещение из 7 элементов по 3 (без повторений).

Пример №44

Найдите количество трехзначных чисел, которые можно составить из цифр 1, 2, 3, 4, 5, 6, 0, если цифры в числе не повторяются.

Выбор формулы проводится таким же образом, как и в задаче 2. Следует учесть, что если число, составленное из трех цифр, начинается цифрой О, то оно не считается трехзначным. Следовательно, для ответов на вопросы задачи можно сначала из заданных 7 цифр записать все числа, состоящие из 3 цифр (см. пример 2), а затем из количества полученных чисел вычесть количество чисел, составленных из трех цифр, но начинающих цифрой 0. В последнем случае мы фактически будем из всех цифр без нуля (их 6) составлять двузначные числа. Тогда их количество равно числу размещений из 6 элементов по 2 (см. решение).

Также можно выполнить непосредственное вычисление, последовательно заполняя три места в трехзначном числе и используя правило произведения. В этом случае удобно сделать рассуждения наглядными, изображая соответствующие разряды в трехзначном числе в виде клеточек, например, так:

  • 6 возможностей
  • 6 возможностей
  • 5 возможностей

Решение:

Уравнения на комбинаторику с решениемКоличество трехзначных чисел, которые можно составить из семи цифр (среди которых нет цифры 0), если цифры в числе не повторяются, равно числу размещений из 7 элементов по 3, то есть Уравнения на комбинаторику с решением

Но среди данных цифр есть цифра 0, с которой не может начинаться трехзначное число. Поэтому из размещений из 7 элементов по 3 необходимо исключить те размещения, в которых первым элементом является цифра 0. Их количество равно числу размещений из 6 элементов по 2, то есть Уравнения на комбинаторику с решениемСледовательно, искомое количество трехзначных чисел равноУравнения на комбинаторику с решением

Пример №45

Решите уравнение Уравнения на комбинаторику с решением

Решение:

Уравнения на комбинаторику с решениемТогда получаем Уравнения на комбинаторику с решениемНа ОДЗ это уравнение равносильно уравнениям:Уравнения на комбинаторику с решением

Уравнения, в запись которых входят выражения, обозначающие количество соответствующих соединений из х элементов, считаются определенными только при натуральных значениях переменной х. В данном случае, чтобы выражение Уравнения на комбинаторику с решениемимело смысл необходимо выбирать натуральные значения Уравнения на комбинаторику с решением(в этом случае Уравнения на комбинаторику с решениемтакже существует и, конечно, Уравнения на комбинаторику с решениемДля преобразования уравнения используем соответствующие формулы:Уравнения на комбинаторику с решением

Перестановки

Перестановкой из п элементов называется любое упорядоченное множество из Уравнения на комбинаторику с решениемэлементов

Напомним, что упорядоченное множество — это такое множество, для которого указано, какой элемент находится на первом месте, какой на втором. какой на Уравнения на комбинаторику с решением

Например, переставляя цифры в числе 236 (там множество цифр уже упорядоченное), можно составить такие перестановки без повторений: (2; 3; 6), (2; 6; 3), (3; 2; 6), (3; 6; 2), (6; 2; 3), (6; 3; 2) — всего 6 перестановок*.

Количество перестановок без повторений из Уравнения на комбинаторику с решениемэлементов обозначается Уравнения на комбинаторику с решением(Р — первая буква французского слова permutation — перестановка). Как видим, Уравнения на комбинаторику с решением

Уравнения на комбинаторику с решениемФактически перестановки без повторений из Уравнения на комбинаторику с решениемэлементов являются размещениями из Уравнения на комбинаторику с решениемэлементов по Уравнения на комбинаторику с решениембез повторений, поэтому Уравнения на комбинаторику с решениемПроизведение 1 • 2 • 3 •. • Уравнения на комбинаторику с решениемобозначается

Уравнения на комбинаторику с решением!. Поэтому полученная формула числа перестановок без повторений из Уравнения на комбинаторику с решениемэлементов может быть записана так:

Уравнения на комбинаторику с решением

*Отметим, что каждая такая перестановка определяет трехзначное число, составленное из цифр 2,3,6 так, что цифры в числе не повторяются.

Например, Уравнения на комбинаторику с решением(что совпадает с соответствующим значением, полученным выше).

С помощью факториалов формулу для числа размещений без повторений

Уравнения на комбинаторику с решением

можно записать в другом виде. Для этого умножим и разделим выражение в формуле (1) на произведение Уравнения на комбинаторику с решениемПолучаем Уравнения на комбинаторику с решением

Следовательно, формула числа размещений без повторений из Уравнения на комбинаторику с решениемэлементов по Уравнения на комбинаторику с решениемможет быть записана так:

Уравнения на комбинаторику с решением

Для того чтобы этой формулой можно было пользоваться при всех значениях Уравнения на комбинаторику с решениемв частности, при Уравнения на комбинаторику с решениемдоговорились считать, что

Уравнения на комбинаторику с решением

Например, по формуле (2) Уравнения на комбинаторику с решением

Обратим внимание, что в тех случаях, когда значение Уравнения на комбинаторику с решением! оказывается очень большим, ответы оставляют записанными с помощью факториалов.

Например,Уравнения на комбинаторику с решением

Примеры решения задач:

Напомним, что для выбора формулы при решении простейших комбинаторных задач достаточно выяснить следующее:

  • — Учитывается ли порядок следования элементов в соединении?
  • — Все ли заданные элементы входят в полученное соединение? Если, например, порядок следования элементов учитывается и все п заданных элементов используются в соединении, то по определению это перестановки из п элементов.

Пример №46

Найдите, сколькими способами можно восемь учащихся построить в колонну по одному.

Решение:

Уравнения на комбинаторику с решениемКоличество способов равно числу перестановок из 8 элементов. То есть Уравнения на комбинаторику с решением

Для выбора соответствующей формулы выясняем ответы на вопросы, приведенные выше. Поскольку порядок следования элементов учитывается и все 8 заданных элементов выбираются, то соответствующие соединения — это перестановки из 8 элементов без повторений. Их количество можно вычислить по формуле.

Пример №47

Найдите количество разных четырехзначных чисел, которые можно составить из цифр 0, 3, 7, 9 (цифры в числе не повторяются).

Решение:

Уравнения на комбинаторику с решениемИз четырех цифр 0, 3, 7, 9, не повторяя заданные цифры, можно получить Уравнения на комбинаторику с решениемперестановок. Перестановки, начинающиеся с цифры 0, не являются записью четырехзначного числа — их количество Уравнения на комбинаторику с решением. Тогда искомое количество четырехзначных чисел равно

Уравнения на комбинаторику с решением

Поскольку порядок следования элементов учитывается и для получения четырехзначного числа надо использовать все элементы, то искомые соединения — это перестановки из 4 элементов. Их количество — Уравнения на комбинаторику с решением. При этом необходимо учесть, что в четырехзначном числе на первом месте не может стоять цифра 0. Таких чисел будет столько, сколько раз мы сможем выполнить перестановки из 3 оставшихся цифр, то есть Уравнения на комбинаторику с решением.

Пример №48

Есть десять книг, из которых четыре — учебники. Сколькими способами можно поставить эти книги на полку так, чтобы все учебники стояли рядом?

Решение:

Уравнения на комбинаторику с решениемСначала будем рассматривать учебники как одну книгу. Тогда на полке надо расставить не 10, а 7 книг. Это можно сделать Уравнения на комбинаторику с решениемспособами. В каждом из полученных наборов книг можно выполнить еще Уравнения на комбинаторику с решениемперестановок учебников. По правилу умножения искомое количество способов равно Уравнения на комбинаторику с решением

Задачу можно решать в два этапа. На первом этапе условно будем считать все учебники за 1 книгу. Тогда получим 7 книг (6 не учебников + 1 условная книга — учебник). Порядок следования элементов учитывается и используются все элементы (поставить на полку необходимо все книги). Следовательно, соответствующие соединения — это перестановки из 7 элементов. Их количество — Уравнения на комбинаторику с решением.

На втором этапе решения будем переставлять между собой только учебники. Это можно сделать Уравнения на комбинаторику с решениемспособами. Поскольку нам надо переставить и учебники, и другие книги, то используем правило произведения.

Сочетания без повторений

Сочетанием без повторений из Уравнения на комбинаторику с решениемэлементов по Уравнения на комбинаторику с решениемназывается любое Уравнения на комбинаторику с решением-элементное подмножество Уравнения на комбинаторику с решением-элементного множества.

Например, из множества Уравнения на комбинаторику с решением> можно составить следующие сочетания без повторений из трех элементов: Уравнения на комбинаторику с решением

Количество сочетаний без повторений из п элементов по к элементов обозначается символом Уравнения на комбинаторику с решением(читается: «Число сочетаний из Уравнения на комбинаторику с решением» или «це из Уравнения на комбинаторику с решением», С — первая буква французского слова combinaison — сочетание). Как видим,Уравнения на комбинаторику с решением

Уравнения на комбинаторику с решениемВыясним, сколько всего можно составить сочетаний без повторений из Уравнения на комбинаторику с решениемэлементов по Уравнения на комбинаторику с решением. Для этого используем известные нам формулы числа размещений и перестановок.

Составление размещения без повторений из Уравнения на комбинаторику с решениемэлементов по Уравнения на комбинаторику с решениемпроведем в два этапа. Сначала выберем Уравнения на комбинаторику с решениемразных элементов из заданного Уравнения на комбинаторику с решением-элементного множества, не учитывая порядок выбора этих элементов (то есть выберем Уравнения на комбинаторику с решением-элементное подмножество из Уравнения на комбинаторику с решением-элементного множества — сочетание без повторений из Уравнения на комбинаторику с решением-элементов по Уравнения на комбинаторику с решением). По нашему обозначению это можно сделать Уравнения на комбинаторику с решениемспособами. После этого полученное множество из к разных элементов упорядочим. Его можно упорядочить Уравнения на комбинаторику с решениемспособами. Получим размещения без повторений из Уравнения на комбинаторику с решениемэлементов по Уравнения на комбинаторику с решением. Следовательно, количество размещений без повторений из Уравнения на комбинаторику с решениемэлементов по Уравнения на комбинаторику с решениемв Уравнения на комбинаторику с решениемраз больше числа сочетаний без повторений из Уравнения на комбинаторику с решениемэлементов по Уравнения на комбинаторику с решением. То есть Уравнения на комбинаторику с решениемОтсюда Уравнения на комбинаторику с решениемУчитывая, что по формуле (2) Уравнения на комбинаторику с решением, получаем Уравнения на комбинаторику с решением

Например, Уравнения на комбинаторику с решениемсовпадает со значением, полученным выше.

Используя формулу (3), можно легко обосновать свойство 1 числа сочетаний без повторений, приведенное в таблице 21.

Уравнения на комбинаторику с решением1) Поскольку Уравнения на комбинаторику с решением

Для того чтобы формулу (4) можно было использовать и при Уравнения на комбинаторику с решением, договорились считать, чтоУравнения на комбинаторику с решением. Тогда по формуле (4) Уравнения на комбинаторику с решением.

Если в формуле (3) сократить числитель и знаменатель наУравнения на комбинаторику с решением, то получим формулу, по которой удобно вычислять Уравнения на комбинаторику с решениемпри малых значениях Уравнения на комбинаторику с решением:

Уравнения на комбинаторику с решением

Например, Уравнения на комбинаторику с решением

Вычисление числа сочетаний без повторений с помощью треугольника Паскаля

Для вычисления числа сочетаний без повторений можно применять формулу (3): Уравнения на комбинаторику с решением, а можно последовательно вычислять соответствующие значения, пользуясь таким свойством:

Уравнения на комбинаторику с решением

Уравнения на комбинаторику с решениемДля обоснования равенства (6) найдем сумму Уравнения на комбинаторику с решениемучитывая, что Уравнения на комбинаторику с решением

Уравнения на комбинаторику с решением, следовательно,

Это равенство позволяет последовательно вычислять значения Уравнения на комбинаторику с решениемс помощью специальной таблицы, которая называется треугольником Паскаля. Если считать, что Уравнения на комбинаторику с решением, то таблица будет иметь следующий вид (табл. 23).

Каждая строка этой таблицы начинается с единицы и заканчивается единицей Уравнения на комбинаторику с решением.

Если какая-либо строка уже заполнена, например, третья, то в четвертой строке надо записать на первом месте единицу. На втором месте запишем число, равное сумме двух чисел третьей строки, стоящих над ним левее и правее (поскольку по формуле (6)Уравнения на комбинаторику с решением.

Уравнения на комбинаторику с решением

На третьем месте запишем число, равное сумме двух следующих чисел третьей строки, стоящих над ним левее и правееУравнения на комбинаторику с решением, и т. д. (а на последнем месте снова запишем единицу).

Примеры решения задач:

Обратим внимание, что, как и раньше, для выбора формулы при решении простейших комбинаторных задач достаточно ответить на вопросы:

  1. Учитывается ли порядок следования элементов в соединении?
  2. Все ли заданные элементы входят в полученное соединение?

Для выяснения того, что заданное соединение является сочетанием, достаточно ответить только на первый вопрос. Если порядок следования элементов не учитывается, то по определению это сочетания из Уравнения на комбинаторику с решениемэлементов по Уравнения на комбинаторику с решениемэлементов.

Пример №49

Из 12 членов туристической группы надо выбрать трех дежурных. Сколькими способами можно сделать этот выбор?

Решение:

Уравнения на комбинаторику с решениемКоличество способов выбрать из 12 туристов трех дежурных равно количеству сочетаний из 12 элементов по 3 (без повторений), то есть

Уравнения на комбинаторику с решением

Для выбора соответствующей формулы выясняем ответы на вопросы, приведенные выше. Поскольку порядок следования элементов не учитывается (для дежурных неважно, в каком порядке их выберут), то соответствующее соединение является сочетанием из 12 элементов по 3 (без повторений). Для вычисления можно использовать формулы (3) или (5), в данном случае применяем формулу (3):Уравнения на комбинаторику с решением

Пример №50

Из вазы с фруктами, в которой лежит 10 разных яблок и 5 разных груш, требуется выбрать 2 яблока и 3 груши. Сколькими способами можно сделать такой выбор?

Решение:

Уравнения на комбинаторику с решениемВыбрать 2 яблока из 10 можно Уравнения на комбинаторику с решениемспособами. При каждом выборе яблок груши можно выбрать способами. Тогда по правилу произведения выбор требуемых фруктов можно выполнить Уравнения на комбинаторику с решениемспособами. Получаем

Уравнения на комбинаторику с решением

Сначала отдельно выберем 2 яблока из 10 и 3 груши из 5. Поскольку при выборе яблок или груш порядок следования элементов не учитывается, то соответствующие соединения — сочетания без повторений.

Учитывая, что требуется выбрать 2 яблока и 3 груши, используем правило произведения и перемножим полученные возможности выбора яблок(Уравнения на комбинаторику с решением) и груш (Уравнения на комбинаторику с решением).

Бином Ньютона

Уравнения на комбинаторику с решением

Поскольку Уравнения на комбинаторику с решениемто формулу бинома Ньютона можно записать еще и так:

Уравнения на комбинаторику с решением

Общий член разложения степени бинома имеет вид Уравнения на комбинаторику с решением

Коэффициенты Уравнения на комбинаторику с решениемназывают биномиальными коэффициентами.

Свойства биномиальных коэффициентов:

  1. Число биномиальных коэффициентов (а следовательно, и число слагаемых в разложении Уравнения на комбинаторику с решениемстепени бинома) равноУравнения на комбинаторику с решением
  2. Коэффициенты членов, равноудаленных от начала и конца разложения, равны между собой (поскольку Уравнения на комбинаторику с решением
  3. Сумма всех биномиальных коэффициентов равна Уравнения на комбинаторику с решениемУравнения на комбинаторику с решением
  4. Сумма биномиальных коэффициентов, стоящих на четных местах, равна сумме биномиальных коэффициентов, стоящих на нечетных местах.
  5. Для вычисления биномиальных коэффициентов можно воспользоваться треугольником Паскаля, в котором вычисления коэффициентов основываются на формуле Уравнения на комбинаторику с решением

Уравнения на комбинаторику с решением

Уравнения на комбинаторику с решением

В каждом ряду по краям стоят единицы, а каждое из остальных чисел равно сумме двух чисел, находящихся над ним справа и слева Например, Уравнения на комбинаторику с решением

Объяснение и обоснование Бинома Ньютона

Двучлен вида а + х также называют биномом. Из курса алгебры известно, что: Уравнения на комбинаторику с решением

Можно заметить, что коэффициенты разложения степени бинома Уравнения на комбинаторику с решениемпри Уравнения на комбинаторику с решениемсовпадают с числами в соответствующей строке треугольника Паскаля. Оказывается, что это свойство выполняется для любого натурального Уравнения на комбинаторику с решениемто есть справедлива формула:

Уравнения на комбинаторику с решением

Формулу (7) называют биномом Ньютона. Правая часть этого равенства называется разложением степени бинома Уравнения на комбинаторику с решениемУравнения на комбинаторику с решениемназывают биномиальными коэффициентами. Общий член разложения степени бинома имеет вид Уравнения на комбинаторику с решением

Уравнения на комбинаторику с решениемОбосновать формулу (7) можно, например, следующим образом.

Если раскрыть скобки в выражении Уравнения на комбинаторику с решениемто есть умножить бином а + х сам на себя Уравнения на комбинаторику с решениемраз, то получим многочлен Уравнения на комбинаторику с решениемстепени относительно переменной х. Тогда результат можно записать так:

Уравнения на комбинаторику с решением

Чтобы найти значение Уравнения на комбинаторику с решениемподставим в обе части равенства (8) вместо х значение 0. Получаем Уравнения на комбинаторику с решениемможем записать:

Уравнения на комбинаторику с решением

Чтобы найти Уравнения на комбинаторику с решениемсначала возьмем производную от обеих частей равенства (8):

Уравнения на комбинаторику с решением

затем, подставив в обе части полученного равенства (9) х = 0, получим: Уравнения на комбинаторику с решениемУчитывая, чтоУравнения на комбинаторику с решениемможем записать: Уравнения на комбинаторику с решениемАналогично, чтобы найти Уравнения на комбинаторику с решениемвозьмем производную от обеих частей равенства (9):

Уравнения на комбинаторику с решением

и, подставив х = 0 в равенство (10), получим Уравнения на комбинаторику с решениемТогда Уравнения на комбинаторику с решениемДругие коэффициенты находят аналогично. Если продифференцировать Уравнения на комбинаторику с решениемраз равенство (8), то получим:

Уравнения на комбинаторику с решением

Подставляя в последнее равенство х = 0, имеем

Уравнения на комбинаторику с решением

Уравнения на комбинаторику с решением

В каждом ряду по краям стоят единицы, а каждое из остальных чисел равно сумме двух чисел, находящихся над ним справа и слева

Умножим обе части равенства (11) на Уравнения на комбинаторику с решениеми найдем коэффициент

Уравнения на комбинаторику с решением. Подставляя найденные значения Уравнения на комбинаторику с решением

1, 2, . Уравнения на комбинаторику с решением) в равенство (8), получаем равенство (7).Уравнения на комбинаторику с решением

Записывая степень двучлена по формуле бинома Ньютона для небольших значений п, биномиальные коэффициенты можно вычислять по треугольнику Паскаля (табл. 25, см. также табл. 24).

Например,Уравнения на комбинаторику с решением

Так как Уравнения на комбинаторику с решениемформулу бинома Ньютона можно записать в виде:

Уравнения на комбинаторику с решением

а учитывая, чтоУравнения на комбинаторику с решением, еще и так:

Уравнения на комбинаторику с решением

Если в формуле бинома Ньютона (12) заменить х на (-х), то получим формулу возведения в степень разности а — х:

Уравнения на комбинаторику с решением. Например, ( Уравнения на комбинаторику с решением(знаки членов разложения чередуются!).

Свойства биномиальных коэффициентов

1. Число биномиальных коэффициентов (а следовательно, и число слагаемых) в разложении Уравнения на комбинаторику с решением-й степени бинома равно Уравнения на комбинаторику с решением+ 1, поскольку разложение содержит все степени х от 0 до Уравнения на комбинаторику с решением(и других слагаемых не содержит).

2. Коэффициенты членов, равноудаленных от начала и конца разложения, равны между собой, посколькуУравнения на комбинаторику с решением

3. Сумма всех биномиальных коэффициентов равна 2″.

Уравнения на комбинаторику с решениемДля обоснования полагаем в равенстве (13) (или в равенстве (7)) значения а = х = 1 и получаем Уравнения на комбинаторику с решением

Например, Уравнения на комбинаторику с решением

4. Сумма биномиальных коэффициентов, стоящих на четных местах, равна сумме биномиальных коэффициентов, стоящих на нечетных местах,

Уравнения на комбинаторику с решениемДля обоснования возьмем в равенстве (13) значения а =1, х = —1. Получаем

Уравнения на комбинаторику с решением

Тогда Уравнения на комбинаторику с решением

Примеры решения задач:

Пример №51

По формуле бинома Ньютона найдите разложение степени Уравнения на комбинаторику с решением

Для нахождения коэффициентов разложения можно использовать треугольник Паскаля или вычислять их по общей формуле. По треугольнику Паскаля коэффициенты равны: 1, 6, 15, 20, 15, б, 1. Учитывая, что при возведении в степень разности знаки членов разложения чередуются, получаем

Уравнения на комбинаторику с решениемДля упрощения записи ответа можно избавиться от иррациональности в знаменателях полученных выражений (см. решение) или сначала учесть, что ОДЗ заданного выражения: х > 0, и тогда Уравнения на комбинаторику с решениемТо есть заданное выражение можно записать так: Уравнения на комбинаторику с решениеми возвести в степень последнее выражение.

Решение:

Уравнения на комбинаторику с решением

Пример №52

В разложении степени Уравнения на комбинаторику с решениемнайти член, содержащий Уравнения на комбинаторику с решением

Решение:

► ОДЗ: Уравнения на комбинаторику с решением> 0. ТогдаУравнения на комбинаторику с решением

Общий член разложения: Уравнения на комбинаторику с решением

По условию член разложения должен содержатьУравнения на комбинаторику с решением, следовательно,

Уравнения на комбинаторику с решением. Отсюда Уравнения на комбинаторику с решением

Тогда член разложения, содержащий Уравнения на комбинаторику с решением, равенУравнения на комбинаторику с решением

На ОДЗ (b > 0) каждое слагаемое в заданном двучлене можно записать как степень с дробным показателем. Это позволит проще записать общий член разложения степениУравнения на комбинаторику с решением: Уравнения на комбинаторику с решением(где Уравнения на комбинаторику с решением= 0, 1, 2, . Уравнения на комбинаторику с решением), выяснить, какой из членов разложения содержит Уравнения на комбинаторику с решением, и записать его.

Чтобы упростить запись общего члена разложения, удобно отметить, чтоУравнения на комбинаторику с решением

Видео:Математика без Ху!ни. Теория вероятностей, комбинаторная вероятность.Скачать

Математика без Ху!ни. Теория вероятностей, комбинаторная вероятность.

Зачем нужна комбинаторика

Для решения задач с использованием классического определения вероятности необходимо знать основные правила и формулы комбинаторики -раздела математики, изучающего методы решения комбинаторных задач — т.е. задач, связанных с подсчетом числа различных комбинаций.

Пусть Уравнения на комбинаторику с решением— элементы конечного множества. Сформулируем два важных правила, часто применяемых при решении комбинаторных задач.

Правило суммы

Если элемент Уравнения на комбинаторику с решениемможет быть выбран Уравнения на комбинаторику с решениемспособами, элемент / Уравнения на комбинаторику с решениемспособами, . элемент Уравнения на комбинаторику с решениемспособами, то выбор одного из элементов Уравнения на комбинаторику с решениемможет быть осуществлен пУравнения на комбинаторику с решениемспособами.

Пример №53

В группе 30 студентов. Известно, что 5 из них на экзамене по математике получили оценку «отлично», 10 — оценку «хорошо», остальные -«удовлетворительно». Сколько существует способов выбрать одного студента, получившего на экзамене оценку «отлично» или «хорошо»?

Решение:

Студент, получивший оценку «отлично» может быть выбранУравнения на комбинаторику с решениемспособами, оценку «хорошо» — Уравнения на комбинаторику с решениемспособами. По правилу суммы существует Уравнения на комбинаторику с решениемспособов выбора одного студента, получившего на экзамене оценку «отлично» или «хорошо». Уравнения на комбинаторику с решением

Правило произведения

Если элемент Уравнения на комбинаторику с решениемможет быть выбран Уравнения на комбинаторику с решениемспособами, после этого элемент Уравнения на комбинаторику с решениемможет быть выбран Уравнения на комбинаторику с решениемспособами после каждого такого выбора элемент Уравнения на комбинаторику с решениемможет быть выбран Уравнения на комбинаторику с решениемспособами, то выбор всех элементов Уравнения на комбинаторику с решениемв указанном порядке может быть осуществлен Уравнения на комбинаторику с решениемспособами.

Пример №54

В группе 30 студентов. Необходимо выбрать старосту, его заместителя и профорга. Сколько существует способов это сделать?

Решение:

Старостой может быть выбран любой из 30 студентов, его заместителем – любой из оставшихся 29, а профоргом – любой из оставшихся 28 студентов, т.е. Уравнения на комбинаторику с решениемПо правилу произведения общее число способов выбора старосты, его заместителя и профорга равно Уравнения на комбинаторику с решением= = 24360 способов. ◄

Пусть дано множество из n различных элементов. Из этого множества могут быть образованы подмножества из m элементов (0 ≤ m ≤n). Например, из 5 элементов a, b, c, d, e могут быть отобраны комбинации по 2 элемента – ab, bc, cd, ba и т.д., по 3 элемента – abc, cbd, cba и т.д.

Если комбинации из n элементов по m отличаются либо составом элементов, либо порядком их расположения (либо и тем и другим), то такие комбинации называют размещениями из n элементов по m. Число размещений из n элементов по m находится по формуле Уравнения на комбинаторику с решениемгде n! равно произведению n первых чисел натурального ряда, т.е. n! = 1·2·…·n.

Пример №55

Сколько можно записать двузначных чисел, используя без повторения цифры от 1 до 5?

Решение:

В данном случае двузначное число является комбинацией из пяти цифр по две цифры. Поскольку числа отличаются как составом входящих в них цифр, так и порядком их расположения, то в данном случае двузначные числа являются размещениями из пяти цифр по две. Число таких размещений

Уравнения на комбинаторику с решениемЕсли комбинации из n элементов по m отличаются только с о с т а в о м элементов (порядок их расположения не имеет значения), то такие комбинации называют сочетаниями из n элементов по m.

Число сочетаний из n элементов по m находится по формуле Уравнения на комбинаторику с решением

Пример №56

Необходимо выбрать в подарок две из пяти имеющихся различных книг. Сколькими способами можно это сделать?

Решение:

Из смысла задачи следует, что порядок выбора книг не имеет значения. Здесь важен только их состав. Поэтому в данном случае комбинации книг представляют собой сочетания из 5 книг по 2. Число таких комбинаций Уравнения на комбинаторику с решениемЕсли в размещениях из n элементов по m некоторые из элементов (или все) могут оказаться одинаковыми, то такие размещения называют размещениями с повторениями из n элементов по m. Число размещений с повторениями равно Уравнения на комбинаторику с решением

Пример №57

Сколько можно записать трехзначных чисел, которые не содержат цифр 0 и 5?

Решение:

В данном случае трехзначное число является комбинацией из восьми цифр (0 и 5 не учитываются) по три цифры. При этом некоторые из цифр (или все) могут повторяться. Поэтому в данном случае трехзначные числа является размещениями с повторениями из восьми цифр по три. Число таких размещений с повторениями Уравнения на комбинаторику с решениемЕсли в сочетаниях из n элементов по m некоторые из элементов (или все) могут оказаться одинаковыми, то такие сочетания называют сочетаниями с повторениями из n элементов по m. Число сочетаний с повторениями равно Уравнения на комбинаторику с решениемгде Уравнения на комбинаторику с решениемопределяется по формуле (1.6).

Пример №58

В почтовом отделении продаются открытки восьми видов. Сколькими способами можно купить в нем три открытки?

Решение:

Учитывая, что порядок выбора открыток не имеет значения, а важен только их состав, причем некоторые из открыток (или все) могут оказаться одинаковыми, искомое число способов находим по формуле числа сочетаний с повторениями Уравнения на комбинаторику с решениемЕсли комбинации из n элементов отличаются только порядком расположения элементов, то такие комбинации называют перестановками из n элементов. Число перестановок из n элементов находится по формуле Уравнения на комбинаторику с решением

Пример №59

Порядок выступления 5 участников конкурса определяется жребием. Сколько различных вариантов жеребьевки при этом возможно?

Решение:

Каждый вариант жеребьевки отличается только порядком участников конкурса, т.е. является перестановкой из 5 элементов. Их число равно Уравнения на комбинаторику с решениемЕсли в перестановках из общего числа n элементов есть k различных элементов, при этом 1-й элемент повторяется Уравнения на комбинаторику с решениемраз, 2-й элемент – Уравнения на комбинаторику с решениемраз, k-й элемент – Уравнения на комбинаторику с решениемраз, причемУравнения на комбинаторику с решением, то такие перестановки называют перестановками с повторениями из n элементов. Число перестановок с повторениями равно Уравнения на комбинаторику с решением

Пример №60

Сколько можно составить шестизначных чисел, состоящих из цифр 3, 5, 7, в которых цифра 3 повторяется 3 раза, цифра 5 – 2 раза, цифра 7 – 1 раз?

Решение:

Каждое шестизначное число отличается от другого порядком следования цифр (причем Уравнения на комбинаторику с решениема их сумма равна 6), т.е. является перестановкой с повторениями из 6 элементов. Их число равно

Уравнения на комбинаторику с решением

Рекомендую подробно изучить предметы:
  1. Теория вероятностей
  2. Математическая статистика
Ещё лекции с примерами решения и объяснением:
  • Классическое определение вероятности
  • Геометрические вероятности
  • Теоремы сложения и умножения вероятностей
  • Формула полной вероятности
  • Математическая обработка динамических рядов
  • Корреляция — определение и вычисление
  • Элементы теории ошибок
  • Методы математической статистики

При копировании любых материалов с сайта evkova.org обязательна активная ссылка на сайт www.evkova.org

Сайт создан коллективом преподавателей на некоммерческой основе для дополнительного образования молодежи

Сайт пишется, поддерживается и управляется коллективом преподавателей

Whatsapp и логотип whatsapp являются товарными знаками корпорации WhatsApp LLC.

Cайт носит информационный характер и ни при каких условиях не является публичной офертой, которая определяется положениями статьи 437 Гражданского кодекса РФ. Анна Евкова не оказывает никаких услуг.

Видео:Комбинаторика. Сочетание. 10 класс.Скачать

Комбинаторика. Сочетание. 10 класс.

Конспект урока на тему «Решение комбинаторных уравнений» (10 класс)

Сочетаниями без повторений занимался еще великий Паскаль. Он предложил специальную таблицу значений сочетаний без повторений.

Значения представлены в табл. которая называется треугольником Паскаля.

Этот треугольник удивительно красив своей математической красотой, и в его числах можно при желании отыскать различные закономерности. Его можно представить несколько иначе – в виде [26]: равнобедренного треугольника (рис. 10).

Уравнения на комбинаторику с решением

Рис. 10. Треугольник Паскаля

Здесь каждое число, кроме единиц на боковых сторонах, является суммой двух чисел, стоящих над ним. Поэтому:

(приводим к общему знаменателю)

(выносим n ! за скобку в знаменателе)

Из этого соотношения и вытекает эффективный способ рекуррентного вычисления значений биномиальных коэффициентов.

Докажем соотношение 1)

Это может использоваться при вычислениях, например, вместо можно вычислить .

Докажем соотношение 2)

Имеется формула, называемая биномом Ньютона, которая использует выражения числа сочетаний с повторениями

где а, b – действительные или комплексные числа.

Коэффициенты называются биномиальными.

Докажем формулу бинома Ньютона по индукции. Доказательство по индукции предполагает:

1) базис индукции – доказательство того, что формула верна для конкретного n , например, для n =1. В нашем случае мы убедились, что формула верна для n =2,3,4. Убедимся, что она верна и для n =1.

2) индукционный шаг. Предполагая, что формула верна для некоторого n , убеждаются, что тогда она верна и для n +1.

3) при истинности шагов 1 и 2 заключают, что формула верна для любого n .

Приступим к индукционному шагу.

Возьмем выражение и получим из него выражение для n +1. Очевидно, что это можно сделать путем умножения на a + b :

Преобразуем полученное выражение:

Для выполнения индукционного шага необходимо показать, что это выражение равно выражению:

Рассмотрим подвыражение выражения (1): и заменим i на i -1.

Получим , т.е. одинаковые коэффициенты перед выражениями , для числа сочетаний в первом и втором подвыражении выражения (1).Это позволит вынести за скобку. Но тогда в не учтен n -й член подвыражения (суммирование идет до n ): тогда, учитывая его, получаем:

Нетрудно видеть, что можно заменить на , кроме того, мы уже доказали, что , поэтому: , что, очевидно, равно выражению:

По индукции получаем, что формула бинома Ньютона верна для любого n .

С использованием бинома Ньютона докажем следствие №1 о количестве подмножеств множества из n элементов:

Рассмотрим следствие №2: .

На использовании бинома Ньютона основано понятие производящей функции – функции, позволяющей получать комбинаторные числа без вычисления факториала:

. Здесь – функция, производящая биномиальные коэффициенты.

При n =1 получаем 1+ x , т.е. (коэффициент перед 1), (коэффициент перед x ).

При n =2 получаем (1+ x ) 2 =1+2 x + x 2 , т.е. и т.д.

Решение комбинаторных уравнений

В комбинаторике тоже могут решаться уравнения, особенностью которых является то, что неизвестная принадлежит множеству натуральных чисел. Например, уравнения вида , xN , где N – множество натуральных чисел или вида:

При решении комбинаторных уравнений часто необходимо уметь выполнять действия с факториалами типа:

Например, в задаче о сравнении пар записей в базе данных из n записей:

, – что и требовалось доказать.

В комбинаторике рассматриваются и другие типовые комбинаторные комбинации, например, разбиения n -элементного множества на k подмножеств, которые называются блоками разбиения. В информатике вычисления на конечных математических структурах часто называют комбинаторными вычислениями, и они требуют комбинаторного анализа для установления свойств и оценки применимости используемых алгоритмов. На рис. 11 приведен один из возможных вариантов классификации основных комбинаций.

Уравнения на комбинаторику с решением

Рис. 11. Основные комбинации

Комбинаторные задачи могут быть решены, например, системой компьютерной математики Matematica (3,4) фирмы Wolfram Research , Inc . – пакет расширения «Дискретная математика» ( DiscreteMath ) – комбинаторика и ее функции ( Combinatorica , CombinatorialFunctions ): функции перестановок и сочетаний и др.

Пример 1. Решить уравнение

Уравнения на комбинаторику с решением

Уравнения на комбинаторику с решением

и представим правую часть в виде

Уравнения на комбинаторику с решением,

Уравнения на комбинаторику с решением

Уравнения на комбинаторику с решением

Уравнения на комбинаторику с решениемоткуда следует

Уравнения на комбинаторику с решением

Уравнения на комбинаторику с решением

x + 3 = 11 и x = 8.

Пример 2. Решить уравнение

Уравнения на комбинаторику с решением

Решение. По условию x – целое число, удовлетворяющее неравенством Уравнения на комбинаторику с решениемПерепишем уравнение в виде

Уравнения на комбинаторику с решением

Уравнения на комбинаторику с решением

Уравнения на комбинаторику с решением

Уравнения на комбинаторику с решением

Уравнения на комбинаторику с решением

Уравнения на комбинаторику с решением

откуда, после упрощений, получаем

Уравнения на комбинаторику с решением

Уравнения на комбинаторику с решением> 4

Пример 3. Решить систему уравнений

Уравнения на комбинаторику с решением

Решение. Из второго уравнение находим

Уравнения на комбинаторику с решениемРешая последнее уравнение, получаем Уравнения на комбинаторику с решениемНо так как Уравнения на комбинаторику с решениемне пригодно к решению уравнения, значит x = 18.

Подставляя x = 18 в первое уравнение системы, найдем

Уравнения на комбинаторику с решением

Уравнения на комбинаторику с решением

18 – y = y + 2, y = 8.

Итак, x = 18, y = 8.

Пример 4. Решить систему уравнений

Уравнения на комбинаторику с решением

Решение. Перепишем систему уравнений в виде

Уравнения на комбинаторику с решениемили, после упрощений получим

Уравнения на комбинаторику с решениемоткуда следует x = 2, y = 6.

Решите уравнение (22–25) .

1)Уравнения на комбинаторику с решением=42;

ОДЗ: хУравнения на комбинаторику с решениемN; x > 2

Уравнения на комбинаторику с решением= 42

Уравнения на комбинаторику с решением

Уравнения на комбинаторику с решением

Уравнения на комбинаторику с решением=-6( исключить – не входит в ОДЗ); Уравнения на комбинаторику с решением=7

Уравнения на комбинаторику с решением=56х;

ОДЗ: хУравнения на комбинаторику с решениемN; x > 3

Уравнения на комбинаторику с решением= Уравнения на комбинаторику с решением

Уравнения на комбинаторику с решением

(Уравнения на комбинаторику с решением

Уравнения на комбинаторику с решением((Уравнения на комбинаторику с решением

Уравнения на комбинаторику с решениемили Уравнения на комбинаторику с решением-3Уравнения на комбинаторику с решением

Уравнения на комбинаторику с решением1 =0(исключить) или х 2 =-6 (исключить); х 3 =9 (входит в ОДЗ).

3)Уравнения на комбинаторику с решением=30;

ОДЗ: хУравнения на комбинаторику с решениемN; x+1 > 2; х > 1

Уравнения на комбинаторику с решением= Уравнения на комбинаторику с решением

Уравнения на комбинаторику с решением

Уравнения на комбинаторику с решением

Уравнения на комбинаторику с решением=-6( исключить – не входит в ОДЗ); Уравнения на комбинаторику с решением=5.

4) 5Уравнения на комбинаторику с решением=Уравнения на комбинаторику с решением;

ОДЗ: Уравнения на комбинаторику с решением Уравнения на комбинаторику с решениемхУравнения на комбинаторику с решением

Уравнения на комбинаторику с решением; Уравнения на комбинаторику с решением=Уравнения на комбинаторику с решением

Уравнения на комбинаторику с решением= Уравнения на комбинаторику с решением

Уравнения на комбинаторику с решением= Уравнения на комбинаторику с решением

(20(х-2)-(х+1)(х+2))Уравнения на комбинаторику с решениемхУравнения на комбинаторику с решением

(20х-40-х 2 +2х+х+2)=0 или х=0 или х-1=0

х 2 +3х-20х+42=0 х 1 =0 х 2 =1

х 2 -17х+42=0 корни 0 и 1 не входят в ОДЗ

Уравнения на комбинаторику с решением= 21 ОДЗ: хУравнения на комбинаторику с решениемN; x-3 > 2 ; x > 3

Уравнения на комбинаторику с решением= Уравнения на комбинаторику с решением

Уравнения на комбинаторику с решением

Уравнения на комбинаторику с решением

Уравнения на комбинаторику с решением— 7х + 12 – 42 = 0

Уравнения на комбинаторику с решением— 7х – 30 = 0

х 1 =10 х 2 = — 3 (не входит в ОДЗ)

2) Уравнения на комбинаторику с решением; ОДЗ: хУравнения на комбинаторику с решениемN; x > 3

Уравнения на комбинаторику с решением= Уравнения на комбинаторику с решением

Уравнения на комбинаторику с решением= Уравнения на комбинаторику с решением

4х(х-2)(х-1) = 6Уравнения на комбинаторику с решением

х(4х 2 – 12х+8-30х+90)=0

х=0 или 4х 2 – 42х + 98 = 0

2х 2 – 21х + 49 = 0

Уравнения на комбинаторику с решением= 15(х-1) ОДЗ: хУравнения на комбинаторику с решениемN; x > 3

Уравнения на комбинаторику с решением= 15(х-1)

Уравнения на комбинаторику с решением= (х-1)х х 1 = 0 или х 2 = 1 — не входят в ОДЗ

Уравнения на комбинаторику с решением= Уравнения на комбинаторику с решениемОДЗ: хУравнения на комбинаторику с решениемN; x > 4

Уравнения на комбинаторику с решением= Уравнения на комбинаторику с решением

4(х-2)! = 24Уравнения на комбинаторику с решением

х 1 =12; х 2 = — 7(не входит в ОДЗ)

Уравнения на комбинаторику с решением= 43 ОДЗ: хУравнения на комбинаторику с решениемN; x > 5

Уравнения на комбинаторику с решением= 43

Уравнения на комбинаторику с решением

х 1 =10; х 2 = 3 (не входит в ОДЗ)

Уравнения на комбинаторику с решением= 89 ОДЗ: хУравнения на комбинаторику с решениемN; x > 7

Уравнения на комбинаторику с решением

х 2 – 11х – 60 = 0

х 1 =15; х 2 = — 4(не входит в ОДЗ)

Уравнения на комбинаторику с решением+ Уравнения на комбинаторику с решением= 162 ОДЗ: хУравнения на комбинаторику с решениемN; x > 1

Уравнения на комбинаторику с решением= 162

Уравнения на комбинаторику с решением= 162

2Уравнения на комбинаторику с решением

24х + х 2 + 7х + 12 – 324 = 0

х 2 + 31х – 312 = 0

х 1 =8; х 2 = — 39(не входит в ОДЗ)

Уравнения на комбинаторику с решением= Уравнения на комбинаторику с решением

ОДЗ: Уравнения на комбинаторику с решениемx > 4

Уравнения на комбинаторику с решением= Уравнения на комбинаторику с решением

Уравнения на комбинаторику с решением= Уравнения на комбинаторику с решением

(х-2)(х-1)х = 0 или (х-3)-45 = 0

х 1 =2; х 2 = 1 х 3 =0 — не входят в ОДЗ х 4 = 48

Уравнения на комбинаторику с решением= 42 ОДЗ: хУравнения на комбинаторику с решениемN; x > 4

Уравнения на комбинаторику с решением= 12

Уравнения на комбинаторику с решением= 12 х 2 – х – 12 = 0 х 1 =4; х 2 = — 3(не входит в ОДЗ) Ответ: 4.

Уравнения на комбинаторику с решением= 90 ОДЗ: Уравнения на комбинаторику с решением Уравнения на комбинаторику с решениемУравнения на комбинаторику с решением

Уравнения на комбинаторику с решением= 90

х 1 =10; х 2 = — 9(не входит в ОДЗ)

Уравнения на комбинаторику с решением= 132 ОДЗ: Уравнения на комбинаторику с решением

Уравнения на комбинаторику с решением= 132

Уравнения на комбинаторику с решением= 132

x 2 +3 x +2–132 = 0

х 1 =10; х 2 = — 13(не входит в ОДЗ)

Уравнения на комбинаторику с решением= 110 ОДЗ: Уравнения на комбинаторику с решением

Уравнения на комбинаторику с решением= 110

Уравнения на комбинаторику с решением= 110

x 2 +3 x +2– 110 = 0

x 2 +3 x – 108 = 0

х 1 =9; х 2 = — 12(не входит в ОДЗ)

Уравнения на комбинаторику с решениемОДЗ: Уравнения на комбинаторику с решением

Уравнения на комбинаторику с решением

Уравнения на комбинаторику с решением

Уравнения на комбинаторику с решением Уравнения на комбинаторику с решениемУравнения на комбинаторику с решением

Уравнения на комбинаторику с решениемрешаем методом сложения — 5у = -30; у = 6

Уравнения на комбинаторику с решениемОДЗ: Уравнения на комбинаторику с решениемУравнения на комбинаторику с решением; уУравнения на комбинаторику с решением

Уравнения на комбинаторику с решением Уравнения на комбинаторику с решениемУравнения на комбинаторику с решением

Уравнения на комбинаторику с решением Уравнения на комбинаторику с решениемУравнения на комбинаторику с решением

Уравнения на комбинаторику с решением

(х-3)(х-2)(х-1) = 3Уравнения на комбинаторику с решением

4) Уравнения на комбинаторику с решением

Уравнения на комбинаторику с решением

Уравнения на комбинаторику с решением

Уравнения на комбинаторику с решением

Уравнения на комбинаторику с решением

Сколько двузначных чисел можно составить из цифр 1. 3, 5, 8, 9 так, чтобы в каждом числе не было одинаковых цифр?

Из 6 открыток надо выбрать 3. Сколькими способами это можно сделать?

Видео:9 класс, 26 урок, Комбинаторные задачиСкачать

9 класс, 26 урок, Комбинаторные задачи

Математика — онлайн помощь

Уравнения на комбинаторику с решением

Рассмотрим множество, состоящее из n различных элементов. Требуется выбрать из них какие-нибудь k элементов и расположить эти k элементов в каком-либо порядке. Такие упорядоченные последовательности называются размещениями из n элементов по k элементов (упорядоченные – следовательно, последовательности и — различные размещения).

Если в последовательности нет одинаковых элементов, то говорят о размещении без повторений. Их количество

Уравнения на комбинаторику с решением

Если в последовательности допускается наличие одинаковых элементов, то говорят о размещении с повторениями. Их количество

Уравнения на комбинаторику с решением

Любое подмножество (неупорядоченное), состоящее из k элементов, называется сочетанием из n элементов по k элементов.

Различные сочетания отличаются друг от друга только самими входящими в них элементами, порядок их следования безразличен, т.е. по условию задачи подмножества и не различны (соединены).

Число сочетаний без повторений

Уравнения на комбинаторику с решением.

Число сочетаний с повторениями

Уравнения на комбинаторику с решением.

Количество способов переставить элементов в заданном множестве (количество перестановок) вычисляется по формуле

Уравнения на комбинаторику с решением.

При решении простейших комбинаторных задач можно использовать следующую таблицу, определяющую число множеств, состоящих из k элементов, отбираемых из множества, содержащего n элементов

ВыборНеупорядоченныйУпорядоченный
Без повтораУравнения на комбинаторику с решениемУравнения на комбинаторику с решением
С повторомУравнения на комбинаторику с решениемУравнения на комбинаторику с решением

Рассмотрим разницу между сочетаниями, размещениями с повторениями, без повторений на следующих примерах.

Видео:02 Комбинаторика ЗадачиСкачать

02  Комбинаторика  Задачи

ПРИМЕРЫ РЕШЕНИЯ ЗАДАЧ

ПРИМЕР 13.2.1 В коробке 6 шаров, пронумерованных от 1 до 6. Из коробки вынимаются друг за другом 3 шара и в этом же порядке записывают полученные цифры. Сколько трехзначных чисел можно таким образом записать?

Решение: По условию задачи подмножества и – различные. Повторов в подмножестве быть не может, так как шары не возвращаются в коробку. Уравнения на комбинаторику с решением

Уравнения на комбинаторику с решением.

ПРИМЕР 13.2.2. В коробке 6 шаров пронумерованных от 1 до 6. Из коробки вынимаются 3 шара и записывают число в порядке возрастания цифр. Сколько трехзначных чисел можно таким образом записать?

Решение: По условию задачи подмножества и дают число 123, т.е. не являются различными.

Уравнения на комбинаторику с решением.

ПРИМЕР 13.2.3. Условие задачи 2.1 (шары возвращаются в коробку)

Решение: Уравнения на комбинаторику с решением.

ПРИМЕР 13.2.4. Условие задачи 2.2 (шары возвращаются в коробку)

Решение: Уравнения на комбинаторику с решением.

ПРИМЕР 13.2.5. Сколько различных перестановок можно составить из букв слова «комар»?

Решение: Уравнения на комбинаторику с решением.

ПРИМЕР 13.2.6. Сколько различных перестановок можно составить из букв слова «задача»?

Решение: Если бы все шесть букв слова были различны, то число перестановок было бы 6! Но буква «а» встречается в данном слове три раза, и перестановки только этих трех букв «а» не дают новых способов расположения букв. Поэтому число перестановок букв слова «задача» будет не 6!, а в 3! раза меньше, то есть Уравнения на комбинаторику с решением.

ПРИМЕР 13.2.7. В мастерской имеется материал 5 цветов. Поступил заказ на пошив флагов, состоящих из трех горизонтальных полос разного цвета каждый. Сколько таких различных флагов может сшить мастерская?

Уравнения на комбинаторику с решением.

Решение: Флаги отличаются друг от друга как цветом полос, так и их порядком, поэтому разных флагов можно сделать Уравнения на комбинаторику с решениемштук.

ПРИМЕР 13.2.8. Сколькими способами можно распределить 5 учеников по 3 параллельным классам?

Решение: Составим вспомогательную таблицу

Номер ученика
Вариант класса

Таким образом, видно, что если для одного ученика существует 3 варианта выбора класса, то для всех 5 учеников существует Уравнения на комбинаторику с решениемспособов распределения по классам.

ПРИМЕР 13.2.9. На книжной полке помещается 30 томов. Сколькими способами их можно расставить, чтобы при этом первый и второй том не стояли рядом?

Решение: Произведем рассуждения “от обратного”. Тридцать томов на одной полке можно разместить 30! способами.

Уравнения на комбинаторику с решением.

Если 1 и 2 тома должны стоять рядом, то число вариантов расстановки сокращается до Уравнения на комбинаторику с решением, т.к. комбинацию из 1 и 2 тома можно считать за один том, но при этом они могут стоять как (1;2) или (2;1), т.е.

Уравнения на комбинаторику с решением, Уравнения на комбинаторику с решением.

Тогда искомое число способов расстановки есть

Уравнения на комбинаторику с решением

ПРИМЕР 13.2.10. Чемпионат, в котором участвуют 16 команд, проводится в два круга, т.е. каждая команда дважды встречается с любой другой. Определить, какое количество встреч следует провести.

Решение: По условию задачи из 16 команд для каждой встречи требуется отобрать 2 команды. В данном случае отбор производится без повтора и порядок отбора не важен, т.е. число вариантов — Уравнения на комбинаторику с решением. Так как команды должны играть дважды число вариантов удваивается, т.е. Уравнения на комбинаторику с решением.

ПРИМЕР 13.2.11. Автомобильная мастерская имеет для окраски 10 основных цветов. Сколькими способами можно окрасить автомобиль, если смешивать от 3 до 7 основных цветов?

Решение: По условию задачи отбор цветов для окраски производится без повтора и порядок отбора не важен, т.е. число вариантов зависит лишь от числа отбираемых для окраски цветов — Уравнения на комбинаторику с решением. Поэтому общее число вариантов есть

Уравнения на комбинаторику с решением.

ПРИМЕР 13.2.12. Турист прошел маршрут из пункта A в пункт B, из B в C и вернулся обратно. Сколько вариантов маршрута существует, если из пункта A в пункт B ведут 3 дороги, а из B в C — 4 и нельзя возвращаться той дорогой, по которой уже прошел?

Решение: Составим схему.

Уравнения на комбинаторику с решением

Из рисунка видно, что вариантов маршрута из А в B существует 3, и из B в C – 4, т.е. всего маршрутов Уравнения на комбинаторику с решением.

Уравнения на комбинаторику с решением

На обратном пути вариантов маршрута из С в B существует 3 (один уже пройден), и из B в А – 2, т.е. всего возможных обратных маршрутов осталось Уравнения на комбинаторику с решением. Тогда всего вариантов маршрута Уравнения на комбинаторику с решением.

ПРИМЕР 13.2.13. Двенадцати ученикам выданы два варианта контрольной работы. Сколькими способами можно посадить учеников в два ряда по 6 человек, чтобы у сидящих рядом не было одинаковых вариантов, а у сидящих друг за другом был один и тот же вариант?

Решение: Рассуждения произведем несколькими способами

I способ) Первоначально 12 учеников разбивают на 2 группы по 6 человек. Это можно сделать Уравнения на комбинаторику с решениемспособами.

Затем они могут распределиться по своим рядам согласно схеме

Уравнения на комбинаторику с решением

Уравнения на комбинаторику с решением.

Поэтому всего способов распределения учеников будет Уравнения на комбинаторику с решением.

II способ) Первоначально 12 учеников запускают в класс, указывая место, где каждый должен сидеть, например “второй ряд, третье место”. Так как посадочных мест также 12, то всего вариантов распределения 12!
Варианты контрольной работы могут распределиться

“I вариант – I ряд, II вариант – II ряд”

“II вариант – I ряд, I вариант – II ряд”,

Таким образом, всего способов распределения учеников будет Уравнения на комбинаторику с решением.

По приведенным решениям видно, что результаты решений совпадают.

ПРИМЕР 13.2.14. Сколько существует вариантов расположения шести гостей за круглым шестиместным столом?

Решение: Эта задача имеет разные решения и, соответственно разные ответы – в зависимости от того, что понимать под различным расположением гостей за столом. Поэтому исследуем возможные варианты.

Если считать, что нам важно, кто сидит на каком стуле, то это простая задача на перестановки и, следовательно, всего вариантов Уравнения на комбинаторику с решением.

Если же важно не то, кто какой стул занял, а то, кто рядом с кем сидит, то требуется рассмотреть варианты взаимного расположения гостей. В таком случае, расположения гостей, получаемые одно из другого при повороте гостей вокруг стола, фактически являются одинаковыми (смотри рисунок).

Очевидно, что для любого расположения гостей таких одинаковых вариантов, получаемых друг из друга поворотом, — шесть. Тогда общее число вариантов уменьшается в шесть раз и их остается Уравнения на комбинаторику с решением.
В случае же, когда нас интересует только взаимное расположение гостей, то одинаковыми можно считать и такие симметричные расположения, при которых у каждого гостя остаются те же соседи за столом, только левый и правый меняются местами (смотри рисунок).

В такой постановке вопроса общее число различных вариантов расположений гостей уменьшается вдвое и составляет 60.

Отметим, что каждое решение будет считаться правильным при соответствующей постановке задачи.

ПРИМЕР 13.2.15. Семнадцать студентов сдали экзамены по 4 предметам только на “хорошо” и “отлично”. Верно ли утверждение, что хотя бы у двух из них оценки по экзаменационным предметам совпадают?

Решение: Очевидно, что в данном случае речь идет о возможных вариантах вида

Предмет1234
Студент 14455
Студент 25445
Студент 35555
Студент 174454

Данный пример можно решить способом, изложенным в примере 13.1.8., и получить количество вариантов Уравнения на комбинаторику с решением. Приведем другой наглядный способ решения, использующий так называемое “дерево решений”,который представляет все варианты (16 штук) получения экзаменационных оценок.

Уравнения на комбинаторику с решением.

По “дереву решений” видно, что 16 студентов могут сдать экзамены только на “хорошо” и “отлично” так, что их результаты будут отличаться, но если студентов 17, хотя бы одно повторение обязательно будет.

При решении задач комбинаторики используются следующие правила.

Если некоторый объект A может быть выбран из совокупности объектов m способами, а другой объект B может быть выбран nспособами, то:

Правило суммы: выбрать либо A, либо B можно m+n способами.

Правило произведения. Пара объектов (A,B) в указанном порядке может быть выбрана Уравнения на комбинаторику с решениемспособами.

Видео:Математика это не ИсламСкачать

Математика это не Ислам

Примеры и задачи для самостоятельного решения

Решить комбинаторную задачу.

13.2.1.1. В группе 25 студентов. Сколькими способами можно выбрать старосту, заместителя старосты и профорга?

13.2.1.2. В группе 25 студентов. Сколькими способами можно выбрать актив группы, состоящий из старосты, заместителя старосты и профорга?

13.2.1.3. Сколькими способами можно составить список из 10 человек?

13.2.1.4. Сколькими способами из 15 рабочих можно создать бригады по 5 человек в каждой?

13.2.1.5. Буквы азбуки Морзе образуются как последовательности точек и тире. Сколько букв можно составить, используя для кодировки каждой из букв: а) ровно 5 символов? б) не более пяти символов?

13.2.1.6. Кости для игры в домино метятся двумя цифрами. Кости симметричны, и поэтому порядок чисел не существенен. Сколько различных костей можно образовать, используя числа 0,1,2,3,4,5,6?

13.2.1.7. Сколько различных звукосочетаний можно взять на десяти выбранных клавишах рояля, если каждое звукосочетание может содержать от трех до десяти различных звуков?

13.2.1.8. В вазе стоят 10 красных и 5 розовых гвоздик. Сколькими способами можно выбрать из вазы пять гвоздик одного цвета?

13.2.1.9. В некоторых странах номера трамвайных маршрутов обозначаются двумя цветными фонарями. Какое количество различных маршрутов можно обозначить, если использовать фонари восьми цветов?

13.2.1.10. Команда компьютера записывается в виде набора из восьми цифровых знаков – нулей и единиц. Каково максимальное количество различных команд?

13.2.1.11. Десять групп занимаются в десяти расположенных подряд аудиториях. Сколько существует вариантов расписания, при которых группы 1 и 2 находились бы в соседних аудиториях?

13.2.1.12. Два почтальона должны разнести 10 писем по 10 адресам. Сколькими способами они могут распределить работу?

13.2.1.13. Замок открывается только в том случае, если набран определенный трехзначный номер. Попытка состоит в том, что набирают наугад три цифры из заданных пяти. Угадать номер удалось только на последней из всех возможных попыток. Сколько попыток предшествовало удачной?

13.2.1.14. Номер автомобильного прицепа состоит из двух букв и четырех цифр. Сколько различных номеров можно составить, используя 30 букв и 10 цифр?

13.2.1.15. У одного студента есть 7 DVD дисков, а у другого – 9 дисков. Сколькими способами они могут обменять 3 диска одного на 3 диска другого?

13.2.1.16. На вершину горы ведут 7 дорог. Сколькими способами турист может два раза подняться на гору и спуститься с нее, если по одной и той же дороге нельзя проходить дважды?

13.2.1.17. У ювелира было 9 разных драгоценных камней: сапфир, рубин, топаз и т.д. Ювелир планировал изготовить браслет для часов, однако три камня было украдено. Насколько меньше вариантов браслета он может изготовить по сравнению с первоначальными планами?

13.2.1.18. В поезд метро на начальной станции вошли 10 пассажиров. Сколькими способами могут выйти все пассажиры на последующих 6 станциях?

13.2.1.19. За одним столом надо рассадить 5 мальчиков и 5 девочек так, чтобы не было двух рядом сидящих мальчиков и двух рядом сидящих девочек. Сколькими способами это можно сделать?

13.2.1.20. В классе 25 учеников. Верно ли утверждение, что, по крайней мере, у трех из них день рождения в один и тот же месяц?

13.2.1.21. На участке железной дороги расположено 25 станций с билетной кассой в каждой. Касса каждой станции продает билеты до любой другой станции, притом в обоих направлениях. Сколько различных вариантов билетов можно выдать на этом участке?

13.2.1.22. На официальном приеме 50 человек обменялись рукопожатиями. Сколько было сделано рукопожатий?

13.2.1.23. Сколько диагоналей у выпуклого двадцатиугольника?

Уважаемые студенты
На нашем сайте можно получить помощь по всем разделам математики и другим предметам:
✔ Решение задач
✔ Выполнение учебных работ
✔ Помощь на экзаменах

💥 Видео

Итоговое повторение «2. Уравнения». Часть 5/5Скачать

Итоговое повторение «2. Уравнения». Часть 5/5

ОСНОВЫ КОМБИНАТОРИКИ Урок 5. Общая схема решения комбинаторных задачСкачать

ОСНОВЫ КОМБИНАТОРИКИ Урок 5. Общая схема решения комбинаторных задач

Комбинаторика. Основные формулы (перестановки, сочетания, размещения) и примеры решения задач.Скачать

Комбинаторика. Основные формулы (перестановки, сочетания, размещения) и примеры решения задач.

Задачи на комбинаторику #1Скачать

Задачи на комбинаторику #1

Решение тригонометрических уравнений. Подготовка к ЕГЭ | Математика TutorOnlineСкачать

Решение тригонометрических уравнений. Подготовка к ЕГЭ | Математика TutorOnline

✓ Комбинаторика. Свойства чисел сочетаний | Ботай со мной #132 | Борис ТрушинСкачать

✓ Комбинаторика. Свойства чисел сочетаний | Ботай со мной #132 | Борис Трушин

Решение биквадратных уравнений. 8 класс.Скачать

Решение биквадратных уравнений. 8 класс.

КОМБИНАТОРИКА для ЧАЙНИКОВ | ОЛИМПИАДНАЯ МАТЕМАТИКАСкачать

КОМБИНАТОРИКА для ЧАЙНИКОВ | ОЛИМПИАДНАЯ МАТЕМАТИКА

ПОГОВОРИМ О КОМБИНАТОРИКЕ РАЗМЕЩЕНИЯ НА ЕГЭ ЧАСТЬ I #математика #егэ #огэ #профильныйегэ #shortsСкачать

ПОГОВОРИМ О КОМБИНАТОРИКЕ РАЗМЕЩЕНИЯ НА ЕГЭ ЧАСТЬ I #математика #егэ #огэ #профильныйегэ #shorts
Поделиться или сохранить к себе: